Re: [obm-l] Arimetica Diofanto

2022-01-28 Por tôpico Gabriel Torkomian
https://artofproblemsolving.com/community/q1h2640462p22841017
Tem no aops

Em sex., 28 de jan. de 2022 10:32, Israel Meireles Chrisostomo <
israelmchrisost...@gmail.com> escreveu:

> 3^x-5^y=2
>
> Em sex., 28 de jan. de 2022 09:53, Esaú Gomes 
> escreveu:
>
>> E qual a equação?
>>
>> On Wed, Jan 26, 2022 at 3:33 PM Israel Meireles Chrisostomo <
>> israelmchrisost...@gmail.com> wrote:
>>
>>> Olá pessoal, recentimente estava estudando e me deparei com uma equação
>>> diofantina.eu tentei resolve-la mas ñ sei se está correta a solução ou
>>> incompleta, vcs poderiam por favor me ajudar a fechar o argumento?ñ quero
>>> outra solução só quero fazer da minha solução uma solução top.Tenho a
>>> impressão que falta alguma coisa.
>>>
>>> --
>>> Esta mensagem foi verificada pelo sistema de antivírus e
>>> acredita-se estar livre de perigo.
>>
>>
>> --
>> Esta mensagem foi verificada pelo sistema de antivírus e
>> acredita-se estar livre de perigo.
>
>
> --
> Esta mensagem foi verificada pelo sistema de antivírus e
> acredita-se estar livre de perigo.

-- 
Esta mensagem foi verificada pelo sistema de antiv�rus e
 acredita-se estar livre de perigo.



[obm-l] Operadores Normais e Invariância

2021-01-14 Por tôpico Gabriel Chalfun
Seja E um espaço vetorial de dimensão finita munido de produto interno.Sendo A 
um operador linear é normal, é verdade que se F  (um subespaço de E) é 
invariante por A, então seu complemento ortogonal F⊥ também invariante por A?


[obm-l] Potência de 2 formada somente por dígitos 1 e 2

2019-11-28 Por tôpico Gabriel Chalfun
Existe um n = 2^k que tem apenas 2 e 1 como dígitos?
-- 
Esta mensagem foi verificada pelo sistema de antiv�rus e
 acredita-se estar livre de perigo.



[obm-l] Coordenadas polares

2019-09-06 Por tôpico Gabriel Lopes
Boa tarde, segue perguntas de uma materia que estou tendo dificuldades para
desenvolver um metodo de resolução.

Como passar para coordenadas polares as seguintes regiões:


0 <=x <=1 , 1 -V (1-x^2) <= y <= 1+V(1-x^2)

e,

V2 <= x <= 2,  0 <= y <= V (4-x^2)

Espero que possam ajudar,

Att. Gabriel

-- 
Esta mensagem foi verificada pelo sistema de antiv�rus e
 acredita-se estar livre de perigo.



[obm-l] Re: [obm-l] Re: [obm-l] Dúvida basica equação polar

2019-09-02 Por tôpico Gabriel Lopes
Caro Ralf, obrigado pela resposta.Para mim ficou confuso pq pensei que a
parametrização do círculo se daria colocando como referencia o novo centro
do mesmo. Quando penso em circulos diferentes , por exemplo residindo em
apenas um quadrante tenho dificuldade de imaginar varrendo todos os pontos
. Vou refletir sobre esses casos pois parecem ser obtidos como vc disse de
fato.

Att.Gabriel

Em Seg, 2 de set de 2019 18:04, Ralph Teixeira  escreveu:

> Bom, vale a pena fazer uma figura primeiro... Fez? Note como este circulo
> estah nos primeiro e quarto quadrantes apenas.
>
> Entao suponho que voce fez as contas e descobriu que r=2cos(teta). No
> quarto quadrante vale -pi/2 onde pi/2 terceiro quadrante). Por isso que -pi/2
> Para ser mas exato, o que aconteceria na equacao r=2cos(teta) para teta
> entre pi/2 e 3pi/2... Tipo, experimente pensar em teta=pi para fazer um
> exemplo. Jogando na equacao, ficaria r=2cos(pi)=-2?!?
>
> Aqui ha duas opcoes:
> a) Alguns livros vao insistir que r>=0 sempre. Neste caso, fica claro que
> pi/2 cosseno fica negativo.
> b) Alguns outros livros sao mais "liberais" e permitem r<0 -- a
> interpretacao seria que quando r eh negativo voce anda na reta que forma
> angulo teta com o eixo x NO SENTIDO OPOSTO. Por exemplo, r=-2 e teta=pi eh
> de fato o ponto (2,0) (voce anda na direcao NEGATIVA do eixo x, mas voce
> anda -2, entao acaba andando para a DIREITA duas unidades). Neste caso,
> colocar teta=pi dah um ponto no circulo sim senhor! Mas, mesmo assim, eu
> usaria apenas -pi/2 ponto (2,0) JAH APARECEU com teta=0, e nao vejo porque conta-lo duas vezes
> (e, dependendo da aplicacao, voce NAO QUER contar cada ponto duas vezes).
>
> Abraco, Ralph.
>
> On Mon, Sep 2, 2019 at 4:55 PM Gabriel Lopes  wrote:
>
>> Boa tarde, tenho uma duvida básica da representação em equação polar do
>> círculo  (x-1)^2 +y^2= 1.
>>
>> Pq os intervalo de teta é de -pi/2 a pi/2 e nao de 0 a 2pi?
>>
>> --
>> Esta mensagem foi verificada pelo sistema de antivírus e
>> acredita-se estar livre de perigo.
>
>
> --
> Esta mensagem foi verificada pelo sistema de antivírus e
> acredita-se estar livre de perigo.

-- 
Esta mensagem foi verificada pelo sistema de antiv�rus e
 acredita-se estar livre de perigo.



[obm-l] Dúvida basica equação polar

2019-09-02 Por tôpico Gabriel Lopes
Boa tarde, tenho uma duvida básica da representação em equação polar do
círculo  (x-1)^2 +y^2= 1.

Pq os intervalo de teta é de -pi/2 a pi/2 e nao de 0 a 2pi?

-- 
Esta mensagem foi verificada pelo sistema de antiv�rus e
 acredita-se estar livre de perigo.



Re: [obm-l]

2019-08-31 Por tôpico Gabriel Lopes
Obrigado a todos, vou investigar as recomendações.

Att.Gabriel

Em Sex, 30 de ago de 2019 17:43, Claudio Buffara 
escreveu:

> Acho que também dá pra se inscrever como aluno de cursos livres. Os
> créditos assim adquiridos poderão ser usados posteriormente caso você
> decida se inscrever no mestrado.
>
> On Fri, Aug 30, 2019 at 4:40 PM Michel Torres  wrote:
>
>> Ola,
>> Até onde eu sei, voce deve se inscrever no programa de verão do IMPA e
>> depois solicitar a inscrição no programa de mestrado.
>> pelo menos era assim.
>> mas se não me engano ( faz muito tempo que não acesso o sistema
>> academico) voce pode se inscrever direto no programa de mestrado mas tem
>> que preencher alguns requisitos.
>>
>> ante de mais nada vc deve se cadastrar no sistema academico mesmo para
>> participar do programa de verão
>>
>> abs
>>
>> On Fri, Aug 30, 2019 at 4:21 PM Gabriel Lopes 
>> wrote:
>>
>>> Prezados, boa tarde.Estou um pouco atrasado na minha formação mas
>>> gostaria de recuperar um pouco desse tempo fazendo materias de mestrado no
>>> IMPA, alguem sabe se isso é possivel e qual seria os procedimentos e pré
>>> -requisitos para tal.
>>>
>>> Att. Gabriel
>>>
>>> --
>>> Esta mensagem foi verificada pelo sistema de antivírus e
>>> acredita-se estar livre de perigo.
>>
>>
>> --
>> Esta mensagem foi verificada pelo sistema de antivírus e
>> acredita-se estar livre de perigo.
>
>
> --
> Esta mensagem foi verificada pelo sistema de antivírus e
> acredita-se estar livre de perigo.

-- 
Esta mensagem foi verificada pelo sistema de antiv�rus e
 acredita-se estar livre de perigo.



[obm-l]

2019-08-30 Por tôpico Gabriel Lopes
Prezados, boa tarde.Estou um pouco atrasado na minha formação mas gostaria
de recuperar um pouco desse tempo fazendo materias de mestrado no IMPA,
alguem sabe se isso é possivel e qual seria os procedimentos e pré
-requisitos para tal.

Att. Gabriel

-- 
Esta mensagem foi verificada pelo sistema de antiv�rus e
 acredita-se estar livre de perigo.



Re: [obm-l] Problema sobre Derivadas

2019-08-30 Por tôpico Gabriel Lopes
Ola, boa tarde. Isso é uma simples aplicação da regra da cadeia.

H'(x) = g'(f (x))*f'(x)

H'(3) = g'(f (3))*f'(3) = g (5) * 3 = 9


Em Sex, 30 de ago de 2019 14:16, Luiz Antonio Rodrigues <
rodrigue...@gmail.com> escreveu:

> Olá, pessoal!
> Boa tarde!
> Tudo bem?
> Estou confuso com o problema abaixo.
> Alguém pode me ajudar?
> Reconheço que tenho falhas graves em Cálculo e aproveito para pedir uma
> indicação de material para estudar.
> Muito obrigado!
>
> Temos duas funções f e g e sabemos que:
>
> f(3)=5
> f'(3)=3
>
> f(4)=2
> f'(4)= -3
>
> f(5)=1
> f'(5)=7
>
> g(3)=4
> g'(3)=5
>
> g(5)=3
> g'(5)=4
>
> Se h(x)=g(f(x)), quanto vale h'(3)?
>
>
> --
> Esta mensagem foi verificada pelo sistema de antivírus e
> acredita-se estar livre de perigo.

-- 
Esta mensagem foi verificada pelo sistema de antiv�rus e
 acredita-se estar livre de perigo.



[obm-l] Re: [obm-l] Re: [obm-l] Obm Nível 2 2017

2019-04-05 Por tôpico Gabriel Lopes
Obrigado, achei meio nebuloso mas vou tentar entender " Então tiraremos n^2
cores idênticas a iniciais e (n-1) as cores da primeira coluna " esse
processo nao consegui entender , tirar n^2 cores e nao ter cor alguma
nao? "(n-1)
conjuntos iguais ao iniciais." E aqui nao seriam n conjuntos? Logo depois
vc fala 2n-1 conjuntos identicos, desculpa mas eu nao consegui
compreender.vou me esforçar para absorver.

Tinha pensando no seguinte , tentar provar  q em um tabuleiro n par, se as
colunas  e linhas tivesse o msm numero de elementos entao a malha seria
xadrez, quem sabe por indução, mas seria uma indução para ordem pares
somente, mas nao ta saindo 

Em Sex, 5 de abr de 2019 17:23, Pedro José  escreveu:

> Boa tarde!
> Caso n seja par está resolvido. Pois, sobrará uma quantidade ímpar de
> casas e portanto não há como serem iguais em quantidade.
>
> Caso n ímpar. Uma das cores prevalecerá. Suponhamos que tenhamos X de uma
> cor e X + k da outra com 2X+k=n^2 e k>0
> Nós temos n^2 formas de tirar uma linha e uma coluna.
> Cada vez que tiramos uma linha e uma coluna, tiramos 2n-1 casas. Para que
> fique igual temos que tirar x da que tem menor quantidade e x+K da que tem
> mais, e 2x+k = 2n-1.
> Vamos fazer a seguinte varredura.
> Para cada coluna vamos varrer todas as n linhasEntão tiraremos n^2 cores
> idênticas a iniciais e (n-1) as cores da primeira coluna , depois n^2
> cores idênticas a iniciais e (n-1) as cores da segunda coluna... Ao final
> tiraremos n conjuntos de cores iguais as n^2 iniciais + (n-1) conjuntos
> iguais ao iniciais.
> Ou seja: (2n-1) conjuntos idênticos ao inicial. O que acarreta em (2n-1) X
> de uma cor e (2n-1) (X+k) de outra apresentando uma diferença de (2n-1)K.
> Para que em todas essas retiradas (uma linha e uma coluna) sobrem cores
> idênticas é necessário se retirar de cada vez x da cor em menor quantidade
> e x + k da maior, onde
> 2x+k = 2n-1, todas as vezes. Há n^2 possibilidades de tirar uma linha e
> uma coluna portanto serão retiradas n^2(2n-1), como já visto, só que n^2*
> (x+k) e n^2(x), o que dá uma diferença de n^2k. Mas pelo outro método dava
> (2n-1)k ==> n^2 =2n-1 ==> n= 1  absurdo, pois n>1.
> Portanto, em alguma retirada sobrarão mais de uma cor que de outra.
>
> Saudações,
> PJMS.
>
> Em qui, 4 de abr de 2019 às 01:06, Gabriel Lopes 
> escreveu:
>
>> Seja *n>1* um inteiro e considere um tabuleiro *nxn*, em que algumas das
>> *n²* casas foram pintadas de pretos, e as restantes foram pintadas de
>> branco. Prove que é possível escolhermos uma das *n²* casas do
>> tabuleiro, de modo que, ao removermos completamente a linha e a coluna que
>> a contém, haja um número diferente de casas pretas e de casas brancas,
>> dentre as *(n-1)².*
>>
>> --
>> Esta mensagem foi verificada pelo sistema de antivírus e
>> acredita-se estar livre de perigo.
>
>
> --
> Esta mensagem foi verificada pelo sistema de antivírus e
> acredita-se estar livre de perigo.

-- 
Esta mensagem foi verificada pelo sistema de antiv�rus e
 acredita-se estar livre de perigo.



[obm-l] Obm Nível 2 2017

2019-04-03 Por tôpico Gabriel Lopes
Seja *n>1* um inteiro e considere um tabuleiro *nxn*, em que algumas das
*n²* casas foram pintadas de pretos, e as restantes foram pintadas de
branco. Prove que é possível escolhermos uma das *n²* casas do tabuleiro,
de modo que, ao removermos completamente a linha e a coluna que a contém,
haja um número diferente de casas pretas e de casas brancas, dentre as
*(n-1)².*

-- 
Esta mensagem foi verificada pelo sistema de antiv�rus e
 acredita-se estar livre de perigo.



[obm-l] Re: [obm-l] [Problema da Balança]

2019-03-26 Por tôpico Gabriel Lopes
Para mim o numero de pesagem mínimal é  n-1, para n maior ou igual a 3,
para se obter tanto o maximo quanto o minimo,( faça indução) .Para obter o
maximo e  depois o mínimo separe o o menor na primeira pesagem e prossiga
para obter o maximo n-1 mais n-2 pesagens, acho q é isso

Em Ter, 26 de mar de 2019 21:14, gilberto azevedo 
escreveu:

> DAdos n ( n maior ou igual do que 2 ) objetos de pesos distintos, prove
> que é possivel determinar qual o mais pesado fazendo 2n - 3 pesagens em uma
> balança de pratos. É esse número mínimo de pesagens que permitem determinar
> o mais leve e o mais pesado ?
>
> --
> Esta mensagem foi verificada pelo sistema de antivírus e
> acredita-se estar livre de perigo.

-- 
Esta mensagem foi verificada pelo sistema de antiv�rus e
 acredita-se estar livre de perigo.



Re: [obm-l] Como calcular?

2018-03-03 Por tôpico Gabriel Tostes
Se B2 for maior que 0 aquele limite vai ser sempre maior que uma constante 
qualquer... O problema ta aí, por isso B2 deve ser 0.

Enviado do meu iPad

> Em 1 de mar de 2018, às 22:18, Douglas Oliveira de Lima 
>  escreveu:
> 
> Então, esse problema é bem interessante, se eu não me engano,
> ele tem sua origem com o matemático indiano Ramanujam, em um
> de seus escritos.
> 
> Mas tem uma solução legal na dissertação do meu camarada Carlos Victor, 
> do PROFMAT,
> veja: https://sca.profmat-sbm.org.br/sca_v2/get_tcc3.php?id=27919 , é o 
> problema de número 49.
> 
> Valeu forte abraço do 
> Douglas Oliveira.
> 
> Em 1 de março de 2018 10:08, Bernardo Freitas Paulo da Costa 
>  escreveu:
>> 2018-03-01 0:56 GMT-03:00 Gabriel Tostes :
>> > Define a sequencia A_(n+1)= [ (A_n)^2 - 1 ] / n (1)
>> > Então A_2= sqrt(1+2A3)=sqrt(1+2(sqrt(1+3A4))... Realimentando sempre 
>> > (substituindo A_n=sqrt(1+ nA_n+1)
>> > vemos que A2 se iguala a x se lim n->oo da raiz 2^(n-2) de An é 0.
>> 
>> Eu não entendi esta afirmação "A2 se iguala a x se lim ... = 0".  Como
>> você mostra isso?  Além do mais, a sequência de raizes podia
>> (podia...) tender a infinito.  Acho que também tem que mostrar que não
>> é o caso.  Enfim, o que você escreveu (pode ser que você queira dizer
>> outra coisa) é que "se o limite é zero, então A2 é finito", mas o que
>> você precisa (para o argumento abaixo) é "se A2 é finito, então o
>> limite é zero".
>> 
>> > Seja An=n+1 + Bn. Bn outra sequencia.
>> 
>> Realmente, essa transformação é mágica.  Eu chutei o limite (usando um
>> computador) e daí calculei os outros termos, vi An = n+1, e fui provar
>> que dava.  O que eu usei foi uma sequência dupla, T(n,m) = raiz(1 +
>> n*raiz(1 + (n+1)*raiz(1 +  raiz(1 + m) ... ))).  Claro que
>> T(n,m+1) > T(n,m), portanto lim T(n,m) existe (ou é infinito).  E para
>> provar que não é infinito eu usei que o limite deveria dar T(n,inf) =
>> n+1, e provei que T(n,m) < n+1 para todo m...
>> 
>> > Então, de 1:
>> > n+2+B_(n+1)=n+2 + 2Bn + ( Bn^2 + 2Bn)/n -> B_(n+1) >= 2Bn, uma inducao 
>> > simples traz que:
>> > Bn>=2^(n-2).B2
>> > Entao o limite quando n vai para o infinito da raiz 2^(n-2) de An eh igual 
>> > a B2, ou seja, B2=0 e
>> > X= A2=3
>> 
>> --
>> Bernardo Freitas Paulo da Costa
>> 
>> --
>> Esta mensagem foi verificada pelo sistema de antivírus e
>> Â acredita-se estar livre de perigo.
>> 
>> 
>> =
>> Instru�ões para entrar na lista, sair da lista e usar a lista em
>> http://www.mat.puc-rio.br/~obmlistas/obm-l.html
>> =
> 
> 
> -- 
> Esta mensagem foi verificada pelo sistema de antivírus e 
> acredita-se estar livre de perigo.

-- 
Esta mensagem foi verificada pelo sistema de antiv�rus e
 acredita-se estar livre de perigo.



Re: [obm-l] Como calcular?

2018-03-03 Por tôpico Gabriel Tostes
Pronto! A gente quer que o limite seja 1 na verdade, nao 0 hahah tava me 
confundindo todo. Mas é só isso. 

> Em 3 de mar de 2018, às 15:28, Gabriel Tostes  escreveu:
> 
> Foi um erro, o que eu quis dizer com isso (mas confundi, esse limite 
> claramente nao vai pra 0 pq An eh maior que 1, eh que A raiz 2^(n-2) de An 
> tem que ser "bem comportada". Nao pode acrescentar muito à A2 quando você for 
> realimentando pra parecer cada vez mais com x, pois ai o limite de x - A2 vai 
> ser 0...
> 
> Enviado do meu iPad
> 
>> Em 1 de mar de 2018, às 10:08, Bernardo Freitas Paulo da Costa 
>>  escreveu:
>> 
>> 2018-03-01 0:56 GMT-03:00 Gabriel Tostes :
>>> Define a sequencia A_(n+1)= [ (A_n)^2 - 1 ] / n (1)
>>> Então A_2= sqrt(1+2A3)=sqrt(1+2(sqrt(1+3A4))... Realimentando sempre 
>>> (substituindo A_n=sqrt(1+ nA_n+1)
>>> vemos que A2 se iguala a x se lim n->oo da raiz 2^(n-2) de An é 0.
>> 
>> Eu não entendi esta afirmação "A2 se iguala a x se lim ... = 0".  Como
>> você mostra isso?  Além do mais, a sequência de raizes podia
>> (podia...) tender a infinito.  Acho que também tem que mostrar que não
>> é o caso.  Enfim, o que você escreveu (pode ser que você queira dizer
>> outra coisa) é que "se o limite é zero, então A2 é finito", mas o que
>> você precisa (para o argumento abaixo) é "se A2 é finito, então o
>> limite é zero".
>> 
>>> Seja An=n+1 + Bn. Bn outra sequencia.
>> 
>> Realmente, essa transformação é mágica.  Eu chutei o limite (usando um
>> computador) e daí calculei os outros termos, vi An = n+1, e fui provar
>> que dava.  O que eu usei foi uma sequência dupla, T(n,m) = raiz(1 +
>> n*raiz(1 + (n+1)*raiz(1 +  raiz(1 + m) ... ))).  Claro que
>> T(n,m+1) > T(n,m), portanto lim T(n,m) existe (ou é infinito).  E para
>> provar que não é infinito eu usei que o limite deveria dar T(n,inf) =
>> n+1, e provei que T(n,m) < n+1 para todo m...
>> 
>>> Então, de 1:
>>> n+2+B_(n+1)=n+2 + 2Bn + ( Bn^2 + 2Bn)/n -> B_(n+1) >= 2Bn, uma inducao 
>>> simples traz que:
>>> Bn>=2^(n-2).B2
>>> Entao o limite quando n vai para o infinito da raiz 2^(n-2) de An eh igual 
>>> a B2, ou seja, B2=0 e
>>> X= A2=3
>> 
>> -- 
>> Bernardo Freitas Paulo da Costa
>> 
>> -- 
>> Esta mensagem foi verificada pelo sistema de antivírus e
>> acredita-se estar livre de perigo.
>> 
>> 
>> =
>> Instruções para entrar na lista, sair da lista e usar a lista em
>> http://www.mat.puc-rio.br/~obmlistas/obm-l.html
>> =

-- 
Esta mensagem foi verificada pelo sistema de antiv�rus e
 acredita-se estar livre de perigo.


=
Instru��es para entrar na lista, sair da lista e usar a lista em
http://www.mat.puc-rio.br/~obmlistas/obm-l.html
=


Re: [obm-l] Como calcular?

2018-03-03 Por tôpico Gabriel Tostes
Foi um erro, o que eu quis dizer com isso (mas confundi, esse limite claramente 
nao vai pra 0 pq An eh maior que 1, eh que A raiz 2^(n-2) de An tem que ser 
"bem comportada". Nao pode acrescentar muito à A2 quando você for realimentando 
pra parecer cada vez mais com x, pois ai o limite de x - A2 vai ser 0...

Enviado do meu iPad

> Em 1 de mar de 2018, às 10:08, Bernardo Freitas Paulo da Costa 
>  escreveu:
> 
> 2018-03-01 0:56 GMT-03:00 Gabriel Tostes :
>> Define a sequencia A_(n+1)= [ (A_n)^2 - 1 ] / n (1)
>> Então A_2= sqrt(1+2A3)=sqrt(1+2(sqrt(1+3A4))... Realimentando sempre 
>> (substituindo A_n=sqrt(1+ nA_n+1)
>> vemos que A2 se iguala a x se lim n->oo da raiz 2^(n-2) de An é 0.
> 
> Eu não entendi esta afirmação "A2 se iguala a x se lim ... = 0".  Como
> você mostra isso?  Além do mais, a sequência de raizes podia
> (podia...) tender a infinito.  Acho que também tem que mostrar que não
> é o caso.  Enfim, o que você escreveu (pode ser que você queira dizer
> outra coisa) é que "se o limite é zero, então A2 é finito", mas o que
> você precisa (para o argumento abaixo) é "se A2 é finito, então o
> limite é zero".
> 
>> Seja An=n+1 + Bn. Bn outra sequencia.
> 
> Realmente, essa transformação é mágica.  Eu chutei o limite (usando um
> computador) e daí calculei os outros termos, vi An = n+1, e fui provar
> que dava.  O que eu usei foi uma sequência dupla, T(n,m) = raiz(1 +
> n*raiz(1 + (n+1)*raiz(1 +  raiz(1 + m) ... ))).  Claro que
> T(n,m+1) > T(n,m), portanto lim T(n,m) existe (ou é infinito).  E para
> provar que não é infinito eu usei que o limite deveria dar T(n,inf) =
> n+1, e provei que T(n,m) < n+1 para todo m...
> 
>> Então, de 1:
>> n+2+B_(n+1)=n+2 + 2Bn + ( Bn^2 + 2Bn)/n -> B_(n+1) >= 2Bn, uma inducao 
>> simples traz que:
>> Bn>=2^(n-2).B2
>> Entao o limite quando n vai para o infinito da raiz 2^(n-2) de An eh igual a 
>> B2, ou seja, B2=0 e
>> X= A2=3
> 
> -- 
> Bernardo Freitas Paulo da Costa
> 
> -- 
> Esta mensagem foi verificada pelo sistema de antivírus e
> acredita-se estar livre de perigo.
> 
> 
> =
> Instruções para entrar na lista, sair da lista e usar a lista em
> http://www.mat.puc-rio.br/~obmlistas/obm-l.html
> =

-- 
Esta mensagem foi verificada pelo sistema de antiv�rus e
 acredita-se estar livre de perigo.


=
Instru��es para entrar na lista, sair da lista e usar a lista em
http://www.mat.puc-rio.br/~obmlistas/obm-l.html
=


Re: [obm-l] Como calcular?

2018-02-28 Por tôpico Gabriel Tostes
Define a sequencia A_(n+1)= [ (A_n)^2 - 1 ] / n (1)
Então A_2= sqrt(1+2A3)=sqrt(1+2(sqrt(1+3A4))... Realimentando sempre 
(substituindo A_n=sqrt(1+ nA_n+1)
vemos que A2 se iguala a x se lim n->oo da raiz 2^(n-2) de An é 0.
Seja An=n+1 + Bn. Bn outra sequencia. Então, de 1:
n+2+B_(n+1)=n+2 + 2Bn + ( Bn^2 + 2Bn)/n -> B_(n+1) >= 2Bn, uma inducao simples 
traz que:
Bn>=2^(n-2).B2
Entao o limite quando n vai para o infinito da raiz 2^(n-2) de An eh igual a 
B2, ou seja, B2=0 e 
X= A2=3

Enviado do meu iPad

> Em 28 de fev de 2018, às 21:53, Anderson Torres 
>  escreveu:
> 
> Em 24 de novembro de 2017 15:25, Fabrício Filho  escreveu:
>> Raiz quadrada de (1+2.Raiz quadrada de (1 + 3.Raiz quadrada de (1 + 4.Raiz
>> quadrada de (1 + 5. Raiz quadrada de (1 +...
> 
> Não me parece fácil sequer definir essa sequência em termos dos
> anteriores. Afinal, se por exemplo
> 
> x = raiz(1+2 raiz(1+3 raiz(1)))
> 
> x é a raiz de um polinômio chato
> 
> (((x^2-1)/2)^2-1)^2/3=1
> 
> E não consigo pensar em uma forma de analisar isso para o caso geral...
> 
> 
>> 
>> --
>> Esta mensagem foi verificada pelo sistema de antivírus e
>> acredita-se estar livre de perigo.
> 
> -- 
> Esta mensagem foi verificada pelo sistema de antivírus e
> acredita-se estar livre de perigo.
> 
> 
> =
> Instruções para entrar na lista, sair da lista e usar a lista em
> http://www.mat.puc-rio.br/~obmlistas/obm-l.html
> =

-- 
Esta mensagem foi verificada pelo sistema de antiv�rus e
 acredita-se estar livre de perigo.


=
Instru��es para entrar na lista, sair da lista e usar a lista em
http://www.mat.puc-rio.br/~obmlistas/obm-l.html
=


[obm-l] Re: [obm-l] Revista para olímpicos (gratuita, online)

2018-02-14 Por tôpico gabriel araujo guedes
Aqui na UFRPE,
também desenvolvemos um jornal para OLímpicos.
O nome é: É matemática,oxente!
Segue o link www.ematematicaoxente.com.br/
Abraços,
GG

Em 5 de fevereiro de 2018 09:44, Tássio Naia  escreveu:

> Salve,
>
> Gostaria de sugerir aos colegas a leitura do Archimede Mathematical
> Journal, um periódico voltado para olímpicos.
>
> http://amj-math.com/
>
> Até,
> Tássio
>
> --
> Esta mensagem foi verificada pelo sistema de antivírus e
> acredita-se estar livre de perigo.

-- 
Esta mensagem foi verificada pelo sistema de antiv�rus e
 acredita-se estar livre de perigo.



Re: [obm-l] Passageiros em fila (probabilidade)

2017-09-01 Por tôpico Gabriel Tostes
Bernardo, existe a bijeicao sim! O que vc falou ta certo, vc so nao percebeu 
que a primeira cadeira nao tem dono! A primeira pessoa escolhe aleatoriamente 
qual cadeira vai sentar. Se a ultima nao sentar na cadeira dele vai sempre 
sentar na primeira cadeira obrigatoriamente(dps tu prova isso ai), eu so n 
provei essas coisas pra encurtar o texto. A bijeicao eh: pra cada arranjo que a 
ultima pessoa nao senta na cadeira ela, a gente inverte as posicoes da ultima 
pessoa com a pessoa que sentou na cadeira dela, as duas sao possiveis e eh uma 
bijeicao! Faz os casos pequenos pra tu entender melhor como essa bijeicao 
funciona. Se eu tivesse um caderno pra escrever seria mais facil mostrar. So q 
escrevendo assim eh mais dificil.

Sent from my iPad

> On Aug 31, 2017, at 8:59 PM, Bernardo Freitas Paulo da Costa 
>  wrote:
> 
>> On Wed, Aug 30, 2017 at 2:30 PM, Gabriel Tostes  wrote:
>> Me mandaram esse problema. Primeiro eu fiz tbm com induçao e etc. Mas como o
>> resultado era mto bonito fui pensar de outra maneira, mais rapida. Vamos la:
>> 
>> No decorrer das pessoas sentando, a ultima nao sentará na cadeira dela
>> somente se uma pessoa ja a tenha ocupado. Porem, para a pessoa que for
>> ocupar a cadeira do ultimo passageiro, damos uma opcao a ela: ou sente na
>> primeira cadeira ou na ultima. No primeiro caso a ultima pessoa sentará na
>> cadeira dela, no segundo nao. Em ambos os casos as cadeiras de todas outras
>> pessoas vao estar definidas e, logo, tem uma bijeicao entre os arranjos em
>> que a ultima pessoa senta na cadeira dela ou nao.  A probabilidade eh,
>> entao, 1/2.
> 
> Ter uma prova bijetiva seria legal.  Mas eu confesso que não entendi o
> seu argumento.  Eu acho que você quis dizer o seguinte:
> - Considere todas as possíveis ocorrências do processo das pessoas
> sentando, chame este conjunto de X.
> - Um elemento x de X é uma correspondência das pessoas com os assentos
> em que elas de fato ficaram.
> - O conjunto X não é equiprovável, pois se por exemplo o "louco" se
> sentar na sua cadeira certa (com probabilidade 1/k, onde k é o número
> de pessoas depois dele na fila), só há UM evento em X que corresponde
> a isso.  Por outro lado, se ele se sentar na cadeira de outra pessoa
> (que não seja a última), a probabilidade dos eventos nestas
> circunstâncias é menor, pois tem que usar um "princípio
> multiplicativo" para cada vez que uma pessoa tiver que escolher uma
> cadeira, se a sua estiver ocupada.
> - Suponha que a última pessoa não se sentou no seu assento marcado.
> Isto define um subconjunto Y de X.  Para cada evento y de Y, existe
> uma pessoa p que se sentou na cadeira do último.  Esta pessoa tem j
> pessoas depois dela.
> - Aqui eu começo a não entender... o que quer dizer a sua "opção" de
> "sentar na primeira cadeira"??? Esta tal primeira cadeira já não está
> ocupada?  Ou você quis dizer "o cara, em vez disso, vai se sentar na
> cadeira que o último se sentou"?
> - De qualquer forma, ao alterar a decisão da pessoa p, vai ocorrer
> também uma mudança das probabilidades dos eventos (pois a "nova"
> cadeira que ele escolheu de fato pertencia a outra pessoa, e esta
> outra pessoa agora vai ter que escolher uma outra cadeira para sentar,
> ...)
> 
> Assim, eu não entendi direito como você constrói a bijeção, e mesmo
> que houvesse uma bijeção, você teria que provar que os eventos postos
> em bijeção tem a mesma probabilidade, o que não é imediato, já que os
> eventos em X não são equiprováveis.
> 
> Abraços,
> -- 
> Bernardo Freitas Paulo da Costa
> 
> -- 
> Esta mensagem foi verificada pelo sistema de antivírus e
> acredita-se estar livre de perigo.
> 
> 
> =
> Instruções para entrar na lista, sair da lista e usar a lista em
> http://www.mat.puc-rio.br/~obmlistas/obm-l.html
> =

-- 
Esta mensagem foi verificada pelo sistema de antiv�rus e
 acredita-se estar livre de perigo.


=
Instru��es para entrar na lista, sair da lista e usar a lista em
http://www.mat.puc-rio.br/~obmlistas/obm-l.html
=


Re: [obm-l] Passageiros em fila (probabilidade)

2017-09-01 Por tôpico Gabriel Tostes
Perdao, talvez a confusao ficou pq eu n explicitei direito oq eh a tal da 
"primeira cadeira". Ela eh a cadeira que pertence a primeira pessoa a sentar, 
mas essa vai sentar aleatoriamente. A gente pode dar a opcao pra pessoa que 
sentaria na cadeira do ultimo pq depois disso a cadeira de todos vao se definir 
a partir dessa opcao, qualquer que seja ela, como eu falei no primeiro email.

Sent from my iPad

> On Aug 31, 2017, at 8:59 PM, Bernardo Freitas Paulo da Costa 
>  wrote:
> 
>> On Wed, Aug 30, 2017 at 2:30 PM, Gabriel Tostes  wrote:
>> Me mandaram esse problema. Primeiro eu fiz tbm com induçao e etc. Mas como o
>> resultado era mto bonito fui pensar de outra maneira, mais rapida. Vamos la:
>> 
>> No decorrer das pessoas sentando, a ultima nao sentará na cadeira dela
>> somente se uma pessoa ja a tenha ocupado. Porem, para a pessoa que for
>> ocupar a cadeira do ultimo passageiro, damos uma opcao a ela: ou sente na
>> primeira cadeira ou na ultima. No primeiro caso a ultima pessoa sentará na
>> cadeira dela, no segundo nao. Em ambos os casos as cadeiras de todas outras
>> pessoas vao estar definidas e, logo, tem uma bijeicao entre os arranjos em
>> que a ultima pessoa senta na cadeira dela ou nao.  A probabilidade eh,
>> entao, 1/2.
> 
> Ter uma prova bijetiva seria legal.  Mas eu confesso que não entendi o
> seu argumento.  Eu acho que você quis dizer o seguinte:
> - Considere todas as possíveis ocorrências do processo das pessoas
> sentando, chame este conjunto de X.
> - Um elemento x de X é uma correspondência das pessoas com os assentos
> em que elas de fato ficaram.
> - O conjunto X não é equiprovável, pois se por exemplo o "louco" se
> sentar na sua cadeira certa (com probabilidade 1/k, onde k é o número
> de pessoas depois dele na fila), só há UM evento em X que corresponde
> a isso.  Por outro lado, se ele se sentar na cadeira de outra pessoa
> (que não seja a última), a probabilidade dos eventos nestas
> circunstâncias é menor, pois tem que usar um "princípio
> multiplicativo" para cada vez que uma pessoa tiver que escolher uma
> cadeira, se a sua estiver ocupada.
> - Suponha que a última pessoa não se sentou no seu assento marcado.
> Isto define um subconjunto Y de X.  Para cada evento y de Y, existe
> uma pessoa p que se sentou na cadeira do último.  Esta pessoa tem j
> pessoas depois dela.
> - Aqui eu começo a não entender... o que quer dizer a sua "opção" de
> "sentar na primeira cadeira"??? Esta tal primeira cadeira já não está
> ocupada?  Ou você quis dizer "o cara, em vez disso, vai se sentar na
> cadeira que o último se sentou"?
> - De qualquer forma, ao alterar a decisão da pessoa p, vai ocorrer
> também uma mudança das probabilidades dos eventos (pois a "nova"
> cadeira que ele escolheu de fato pertencia a outra pessoa, e esta
> outra pessoa agora vai ter que escolher uma outra cadeira para sentar,
> ...)
> 
> Assim, eu não entendi direito como você constrói a bijeção, e mesmo
> que houvesse uma bijeção, você teria que provar que os eventos postos
> em bijeção tem a mesma probabilidade, o que não é imediato, já que os
> eventos em X não são equiprováveis.
> 
> Abraços,
> -- 
> Bernardo Freitas Paulo da Costa
> 
> -- 
> Esta mensagem foi verificada pelo sistema de antivírus e
> acredita-se estar livre de perigo.
> 
> 
> =
> Instruções para entrar na lista, sair da lista e usar a lista em
> http://www.mat.puc-rio.br/~obmlistas/obm-l.html
> =

-- 
Esta mensagem foi verificada pelo sistema de antiv�rus e
 acredita-se estar livre de perigo.


=
Instru��es para entrar na lista, sair da lista e usar a lista em
http://www.mat.puc-rio.br/~obmlistas/obm-l.html
=


Re: [obm-l] Passageiros em fila (probabilidade)

2017-08-30 Por tôpico Gabriel Tostes
Me mandaram esse problema. Primeiro eu fiz tbm com induçao e etc. Mas como o 
resultado era mto bonito fui pensar de outra maneira, mais rapida. Vamos la:
No decorrer das pessoas sentando, a ultima nao sentará na cadeira dela somente 
se uma pessoa ja a tenha ocupado. Porem, para a pessoa que for ocupar a cadeira 
do ultimo passageiro, damos uma opcao a ela: ou sente na primeira cadeira ou na 
ultima. No primeiro caso a ultima pessoa sentará na cadeira dela, no segundo 
nao. Em ambos os casos as cadeiras de todas outras pessoas vao estar definidas 
e, logo, tem uma bijeicao entre os arranjos em que a ultima pessoa senta na 
cadeira dela ou nao.  A probabilidade eh, entao, 1/2.

> On Aug 30, 2017, at 2:31 AM, Marcelo Salhab Brogliato  
> wrote:
> 
> Eu vi esse problema no Quora e cheguei na resposta de 1/2, para qualquer 
> tamanho de fila com n>=2. Achei muito interessante! Resolvi por recorrência 
> e indução finita.
> 
> 
> There are 100 people waiting in line to board an airliner with 100 seats.
> 
> The seats are numbered from 1 to 100. Each passenger holds a ticket with his 
> seat number.
> 
> You are the last passenger in line.
> 
> One of the passengers ahead of you in line is crazy. We don’t know which 
> one. He will ignore his seat assignment and sit in a random empty seat.
> 
> Every other passenger will sit in their assigned seat — unless it is 
> already taken, in which case they too will sit in a random empty seat.
> 
> Passengers board one by one.
> 
> What is the probability that you will sit in your assigned seat?
> 
> EDIT for clarification: the crazy person might sit in his assigned seat; he 
> is equally likely to sit in any open seat.
> 
> Link: What is your favorite math problem/puzzle? by John Coiner 
> https://www.quora.com/What-is-your-favorite-math-problem-puzzle/answer/John-Coiner?share=9765a714&srid=vb7t
> 
> -- msbrogli
> 
> Enviado do meu iPhone
> 
> -- 
> Esta mensagem foi verificada pelo sistema de antivírus e 
> acredita-se estar livre de perigo.

-- 
Esta mensagem foi verificada pelo sistema de antiv�rus e
 acredita-se estar livre de perigo.



Re: [obm-l] Re: [obm-l] Questão

2017-08-25 Por tôpico Gabriel Tostes
Faltou so uma coisa, a ordem de 10 mod 23 é 11 nao 22. Entao o k= 2+11k 


> On Aug 25, 2017, at 12:28 AM, Daniel da Silva  
> wrote:
> 
> Obrigado Pedro.
> 
> Daniel Rocha da Silva
> 
> Em 23 de ago de 2017, às 19:31, Pedro José  escreveu:
> 
>> Boa noite!
>> 
>> O difícil é achar o n.
>> 
>> Como o menor inteiro positivo que atende 10^a = 1 mod23 é a=22
>> 
>> E como 10^3 = 11 mod23.
>> 
>> Temos que K + 1 = 3 +22*m com m natural
>> então k = 2 + 22*m.
>> 
>> e n/2 = [10^(k+1) -11]/23 ==> n=2*[10^(k+1)-11]/23.
>> 
>> Portanto as soluções serão (2+ 22*m; 2*[10^(3+22*m)-11]/23; com m= 
>> 0,1, 2, 3, 4
>> 
>> Então há uma infinidade de soluções. você achou a relativa a 
>> m=0.
>> 
>> ou seja, k= 2 e n = 2*[10^3-11]/23=2*43=86
>> 
>> para m =1; k= 24 e n= 869,575.217.391.304.347.826.086
>> 
>> Salvo engano para n pois fiz na marra.
>> 
>> Saudações,
>> PJMS
>> 
>> Em 23 de agosto de 2017 17:19, Daniel da Silva 
>>  escreveu:
>>> Boa tarde,
>>> 
>>> Como saber quantos valores inteiros
>>> de N e K satisfazem a seguinte equação:
>>> 
>>> 10^(K+1)=11+23N/2
>>> 
>>> Encontrei uma solução (N=86, K=2), mas como saber se é 
>>> única?
>>> 
>>> Obrigado,
>>> Daniel Rocha da Silva
>>> --
>>> Esta mensagem foi verificada pelo sistema de antivírus e
>>> Â acredita-se estar livre de perigo.
>>> 
>>> 
>>> =
>>> Instruções para entrar na lista, sair da lista e usar a lista em
>>> http://www.mat.puc-rio.br/~obmlistas/obm-l.html
>>> =
>> 
>> 
>> -- 
>> Esta mensagem foi verificada pelo sistema de antivírus e 
>> acredita-se estar livre de perigo.
> 
> -- 
> Esta mensagem foi verificada pelo sistema de antivírus e 
> acredita-se estar livre de perigo.

-- 
Esta mensagem foi verificada pelo sistema de antiv�rus e
 acredita-se estar livre de perigo.



Re: [obm-l] Re: [obm-l] Questão

2017-08-25 Por tôpico Gabriel Tostes
Confundi, eh 22 msm. :D

> On Aug 25, 2017, at 12:28 AM, Daniel da Silva  
> wrote:
> 
> Obrigado Pedro.
> 
> Daniel Rocha da Silva
> 
> Em 23 de ago de 2017, às 19:31, Pedro José  escreveu:
> 
>> Boa noite!
>> 
>> O difícil é achar o n.
>> 
>> Como o menor inteiro positivo que atende 10^a = 1 mod23 é a=22
>> 
>> E como 10^3 = 11 mod23.
>> 
>> Temos que K + 1 = 3 +22*m com m natural
>> então k = 2 + 22*m.
>> 
>> e n/2 = [10^(k+1) -11]/23 ==> n=2*[10^(k+1)-11]/23.
>> 
>> Portanto as soluções serão (2+ 22*m; 2*[10^(3+22*m)-11]/23; com m= 
>> 0,1, 2, 3, 4
>> 
>> Então há uma infinidade de soluções. você achou a relativa a 
>> m=0.
>> 
>> ou seja, k= 2 e n = 2*[10^3-11]/23=2*43=86
>> 
>> para m =1; k= 24 e n= 869,575.217.391.304.347.826.086
>> 
>> Salvo engano para n pois fiz na marra.
>> 
>> Saudações,
>> PJMS
>> 
>> Em 23 de agosto de 2017 17:19, Daniel da Silva 
>>  escreveu:
>>> Boa tarde,
>>> 
>>> Como saber quantos valores inteiros
>>> de N e K satisfazem a seguinte equação:
>>> 
>>> 10^(K+1)=11+23N/2
>>> 
>>> Encontrei uma solução (N=86, K=2), mas como saber se é 
>>> única?
>>> 
>>> Obrigado,
>>> Daniel Rocha da Silva
>>> --
>>> Esta mensagem foi verificada pelo sistema de antivírus e
>>> Â acredita-se estar livre de perigo.
>>> 
>>> 
>>> =
>>> Instruções para entrar na lista, sair da lista e usar a lista em
>>> http://www.mat.puc-rio.br/~obmlistas/obm-l.html
>>> =
>> 
>> 
>> -- 
>> Esta mensagem foi verificada pelo sistema de antivírus e 
>> acredita-se estar livre de perigo.
> 
> -- 
> Esta mensagem foi verificada pelo sistema de antivírus e 
> acredita-se estar livre de perigo.

-- 
Esta mensagem foi verificada pelo sistema de antiv�rus e
 acredita-se estar livre de perigo.



Re: [obm-l] Algebra (Polinomios)

2017-07-10 Por tôpico Gabriel Tostes
Substitui x+1 por Y. Fica bem na cara, só abrir (y^2-y+1)^40 e ver o que tem 
grau menor que 3. Que é
1-40y+820y^2. Substitui agora denovo e o resto é 
1-40(x+1)+820(x+1)^2=820x^2+1600x+781



Sent from my iPad
> On Jul 10, 2017, at 8:37 PM, Douglas Oliveira de Lima 
>  wrote:
> 
> Encontrar o resto da divisão do polinomio (x^2+x+1)^40 por (x+1)^3.
> 
> Obs: Sem usar derivadas.
> 
> Douglas Oliveira.
> 
> -- 
> Esta mensagem foi verificada pelo sistema de antivírus e 
> acredita-se estar livre de perigo.

-- 
Esta mensagem foi verificada pelo sistema de antiv�rus e
 acredita-se estar livre de perigo.


=
Instru��es para entrar na lista, sair da lista e usar a lista em
http://www.mat.puc-rio.br/~obmlistas/obm-l.html
=


Re: [obm-l] Desigualdade

2017-05-26 Por tôpico Gabriel Tostes
Tira ln, esse produto vai ser: 
Sum{n>=1} ln(n+1)/(2^n) = M

Bora escrever M de outro jeito:

M= ln(2) + [ln(3)-ln(2)]/2 + [ln(4)-ln(3)]/2^2 + ...

M= Sum{n>=1} (ln(n+1)-ln(n))/2^(n-1)

Como ln(n+1)-ln(n)=ln(1+1/n)<1/n

M=2} 1/n.2^(n-1) = L + ln(2)

Para achar L considere:
1/(1-x)= 1+x^2+x^3+...

Integrando essa expressao temos que -(1/x).ln(1-x)= 1+x/2+x^2/3+...
Substituindo x=1/2 achamos que L=2ln(2)-1
E entao
M< 3ln(2)-1 < ln(3)

 E o produto pedido inicialmente eh menor que 3








Sent from my iPad
> On May 26, 2017, at 9:47 PM, Douglas Oliveira de Lima 
>  wrote:
> 
> Como posso fazer essa daqui:
> 
> [2^(1/2)].[3^(1/4)].[4^(1/8)].[5^(1/16)]...<3
> 
> Grande abraço a todos
> 
> DouglasOliveira
> 
> -- 
> Esta mensagem foi verificada pelo sistema de antivírus e 
> acredita-se estar livre de perigo.

-- 
Esta mensagem foi verificada pelo sistema de antiv�rus e
 acredita-se estar livre de perigo.


=
Instru��es para entrar na lista, sair da lista e usar a lista em
http://www.mat.puc-rio.br/~obmlistas/obm-l.html
=


Re: [obm-l] desigualdade

2017-04-30 Por tôpico Gabriel Tostes
Nem vi a condição de q era positivo, de fato n vale.

Sent from my iPad

> On Apr 30, 2017, at 3:53 PM, Douglas Oliveira de Lima 
>  wrote:
> 
> Observe quando x=2, y=3 e z=1 a desigualdade não funciona, logo não basta 
> substituir x+y=a, 
> x+z=b e y+z=c, na verdade acho que  funciona ao "contrário" x/(x+y) + y/ 
> (y+z) + z/(z+x) <= 2.
> A não ser que seja outra questão como por exemplo:
> (x+y)/z +(x+z)/y +(y+z)/x >=6 o que daria certo.
> 
> Grande abraço
> 
> Douglas Oliveira.
> 
> Em 30 de abril de 2017 10:46, marcone augusto araújo borges 
>  escreveu:
>> Se x, y, z são números positivos, prove que x/(x+y) + y/ (y+z) + z/(z+x) > 
>> = 2
>> 
>> 
>> -- 
>> Esta mensagem foi verificada pelo sistema de antivírus e 
>> acredita-se estar livre de perigo.
> 
> 
> -- 
> Esta mensagem foi verificada pelo sistema de antivírus e 
> acredita-se estar livre de perigo.

-- 
Esta mensagem foi verificada pelo sistema de antiv�rus e
 acredita-se estar livre de perigo.



Re: [obm-l] desigualdade

2017-04-30 Por tôpico Gabriel Tostes
X+y=a x+z=b y+z=c so fazer essa substituicao

Sent from my iPad

> On Apr 30, 2017, at 10:46 AM, marcone augusto araújo borges 
>  wrote:
> 
> Se x, y, z são números positivos, prove que x/(x+y) + y/ (y+z) + z/(z+x) > = 2
> 
> -- 
> Esta mensagem foi verificada pelo sistema de antivírus e 
> acredita-se estar livre de perigo.

-- 
Esta mensagem foi verificada pelo sistema de antiv�rus e
 acredita-se estar livre de perigo.



Re: [obm-l] Menor caminho

2017-03-04 Por tôpico Gabriel Tostes
Reflete A nas abcissas e B nas ordenadas e traça linha reta entre eles

> On Mar 4, 2017, at 21:39, Guilherme Oliveira 
>  wrote:
> 
> Considere quatro pontos em um plano cartesiano: A (3,13) e B (9,3). Qual é 
> o caminho de menor comprimento que tenha como extremos os pontos A e B e 
> tenha pelo menos um ponto no eixo das abscissas e outro no eixo das 
> ordenadas? Qual é o seu comprimento?
> 
> 
> 
> --Â 
> __
> 
> “A mente que se abre a uma nova ideia jamais voltará ao seu tamanho 
> original.”
> 
> Â 
> 
> Albert Einstein
> 
> 
> -- 
> Esta mensagem foi verificada pelo sistema de antivírus e 
> acredita-se estar livre de perigo.

-- 
Esta mensagem foi verificada pelo sistema de antiv�rus e
 acredita-se estar livre de perigo.



Re: [obm-l] infinitas soluções(inteiros)

2017-03-03 Por tôpico Gabriel Tostes
ISL 1997 NT 6.
Da pra generalizar ainda x^a+y^b=z^c se Mdc(a,c) ou Mdc(b,c) é 1 e mdc(a,b)=1

Sent from my iPad
> On Mar 3, 2017, at 16:22, marcone augusto araújo borges 
>  wrote:
> 
> Prove que a equação x^1991 + y^1992 = z^1993 tem infinitas soluções x, y, z
> 
> nos inteiros positivos?
> 
> 
> -- 
> Esta mensagem foi verificada pelo sistema de antivírus e 
> acredita-se estar livre de perigo.

-- 
Esta mensagem foi verificada pelo sistema de antiv�rus e
 acredita-se estar livre de perigo.



Re: [obm-l] Re: [obm-l] Cálculo de determinante.

2017-02-27 Por tôpico Gabriel Tostes
Na verdade é um produtorio... Com phi de euler no meio

> On Feb 27, 2017, at 19:54, Anderson Torres  
> wrote:
> 
> Isso já foi respondido em uma Eureka!
> E do que me lembre, não era uma potência de dois não.
> 
> Em 22 de fevereiro de 2017 23:34, Douglas Oliveira de Lima
>  escreveu:
>> Olá caros amigos não consegui pensar no seguinte problema:
>> 
>> 1) Calcular o determinante de uma matriz quadrada de ordem n , onde cada
>> elemento é o MDC entre i e j.
>> 
>> Obs: O resultado é MT bonito, uma potência de 2.
>> 
>> Agradeço a ajuda.
>> 
>> Douglas Oliveira.
>> 
>> 
>> --
>> Esta mensagem foi verificada pelo sistema de antivírus e
>> acredita-se estar livre de perigo.
> 
> -- 
> Esta mensagem foi verificada pelo sistema de antivírus e
> acredita-se estar livre de perigo.
> 
> 
> =
> Instruções para entrar na lista, sair da lista e usar a lista em
> http://www.mat.puc-rio.br/~obmlistas/obm-l.html
> =

-- 
Esta mensagem foi verificada pelo sistema de antiv�rus e
 acredita-se estar livre de perigo.


=
Instru��es para entrar na lista, sair da lista e usar a lista em
http://www.mat.puc-rio.br/~obmlistas/obm-l.html
=


[obm-l] Re: [obm-l] qual destes é o maior?

2017-02-10 Por tôpico Gabriel Lopes
Observe primeiramente que :

# Para todo N>5, natural, temos que:

N! > N*(2^n)  ,por indução.

#Dessa forma:

(2^30)!  < [(2^30)]^(2^30) = 2^[30*(2^30)] < 2^[N!]

Acho que é isso.

-- 
Esta mensagem foi verificada pelo sistema de antiv�rus e
 acredita-se estar livre de perigo.



Re: [obm-l] Aritmética(divisores)

2017-01-12 Por tôpico Gabriel Tostes
Se d15<=n/3 chega num absurdo pela primeira condicao. Entao d15=n/2 e tem 16 
divisores.
Se d14<=n/4 chega num absurdo tambem pela primeira e logo d14=n/3.
Substituindo esses valores em n=d13+d14+d15 achamos que d13=n/6
Entao 2||n e 3|n vamos dividir em dois casos. 3||n e 3^b||n, b>1
1°:
3||n
Da segunda condicao > d15=d5(d5^2+3d5+3) como 3|d15 e 3 nao divide d5 chegamos 
a um absurdo.
2°:
Como uma potencia de 3 maior que um divide n e ja vimos que d5 tem que ser 
multiplo de 3 entao d5=9 e logo n=1998. 



> On Jan 12, 2017, at 16:58, marcone augusto araújo borges 
>  wrote:
> 
> Os divisores de um número inteiro e positivo n estão escritos em ordem 
> crescente a partir
> 
> do número 1 < d1 < d2 < d3 < ... < n.Encontrar o número n sabendo que:
> 
> 
> a) n = d13 + d14 + d15 e
> 
> 
> b) (d5 + 1)^3 = d15 + 1
> 
> 
> Alguém resolveria?
> 
> 
> -- 
> Esta mensagem foi verificada pelo sistema de antivírus e 
> acredita-se estar livre de perigo.

-- 
Esta mensagem foi verificada pelo sistema de antiv�rus e
 acredita-se estar livre de perigo.



Re: [obm-l] Combinatoria

2016-12-26 Por tôpico Gabriel Tostes
Na vdd acho que do jeito que foi feito o andamento da solucao nao precisa 
provar que 31 eh impossivel, ja ta provado.

> On Dec 26, 2016, at 11:04, Bruno Visnadi  wrote:
> 
> Considerando os dígitos (0, 1, 2, 3, 4, 5, 6, 7), temos duas possibilidades:
> Ou dois dos cadeados tem algum dos dígitos (0, 1, 2, 3), ou dois deles tem 
> algum dos dígitos (4, 5, 6, 7).
> Vamos supor que ao menos 2 deles tenham dígitos do grupo (0, 1, 2, 3). 
> Então queremos cobrir todas as possibilidades que envolvem estes dígitos. 
> Cada tentativa (A, B, C) elimina 3 possibilidades (A, B, X), (A, X, C) e (X, 
> B, C). Existem, ao todo, 48 possibilidades (16 pares possíveis, e 3 
> posições). Portanto precisamos de 48/3 = 16 tentativas, desde que não se 
> repita nenhum par. E isto é possível:
> (0, 0, 0), (0, 1, 1), (0, 2, 2), (0, 3, 3), (1, 0, 1), (1, 1, 2), (1, 2, 3), 
> (1, 3, 0), (2, 0, 2), (2, 1, 3), (2, 2, 0), (2, 3, 1), (3, 0, 3), (3, 1, 0), 
> (3, 2, 1), (3, 3, 2).
> Se nossa suposição estava errada, e na verdade 2 dos cadeados tem dígitos 
> do grupo (4, 5, 6, 7), basta fazer as mesmas tentativas, mas trocando os 
> dígitos (0, 1, 2, 3) por (4, 5, 6, 7). Assim, abriremos o armário em no 
> máximo 32 tentativas.
> 
> Em 26 de dezembro de 2016 08:26, Pedro José  escreveu:
>> Bom dia!
>> 
>> Fui inocente, fiz uma restrição que não precisava. Não há necessidade 
>> de acaso.
>> Pode haver estratégia.
>> 
>> Saudações,
>> PJMS
>> 
>> Em 25 de dezembro de 2016 12:31, Matheus Herculano 
>>  escreveu:
>>> 87
>>> 
>>> Em 23 de dez de 2016 13:07, "Gabriel Tostes"  escreveu:
>>>> Um armario de segurança tem 3 cadeados. Cada cadeado tem 8 combinacoes 
>>>> diferentes. O armario abre se quaisquer 2 dos 3 cadeados estao na posicao 
>>>> correta, qual e o numero minimo de tentativas pra abrir o armario?
>>>> 
>>>> 
>>>> --
>>>> Esta mensagem foi verificada pelo sistema de antivírus e
>>>> Â acredita-se estar livre de perigo.
>>>> 
>>>> 
>>>> =
>>>> Instruções para entrar na lista, sair da lista e usar a lista em
>>>> http://www.mat.puc-rio.br/~obmlistas/obm-l.html
>>>> =
>>> 
>>> -- 
>>> Esta mensagem foi verificada pelo sistema de antivírus e 
>>> acredita-se estar livre de perigo.
>> 
>> 
>> -- 
>> Esta mensagem foi verificada pelo sistema de antivírus e 
>> acredita-se estar livre de perigo.
> 
> 
> -- 
> Esta mensagem foi verificada pelo sistema de antivírus e 
> acredita-se estar livre de perigo.

-- 
Esta mensagem foi verificada pelo sistema de antiv�rus e
 acredita-se estar livre de perigo.



Re: [obm-l] Combinatoria

2016-12-26 Por tôpico Gabriel Tostes
Mto bom! So que tem que provar ainda que 31 nao eh possivel :/ 

> On Dec 26, 2016, at 11:04, Bruno Visnadi  wrote:
> 
> Considerando os dígitos (0, 1, 2, 3, 4, 5, 6, 7), temos duas possibilidades:
> Ou dois dos cadeados tem algum dos dígitos (0, 1, 2, 3), ou dois deles tem 
> algum dos dígitos (4, 5, 6, 7).
> Vamos supor que ao menos 2 deles tenham dígitos do grupo (0, 1, 2, 3). 
> Então queremos cobrir todas as possibilidades que envolvem estes dígitos. 
> Cada tentativa (A, B, C) elimina 3 possibilidades (A, B, X), (A, X, C) e (X, 
> B, C). Existem, ao todo, 48 possibilidades (16 pares possíveis, e 3 
> posições). Portanto precisamos de 48/3 = 16 tentativas, desde que não se 
> repita nenhum par. E isto é possível:
> (0, 0, 0), (0, 1, 1), (0, 2, 2), (0, 3, 3), (1, 0, 1), (1, 1, 2), (1, 2, 3), 
> (1, 3, 0), (2, 0, 2), (2, 1, 3), (2, 2, 0), (2, 3, 1), (3, 0, 3), (3, 1, 0), 
> (3, 2, 1), (3, 3, 2).
> Se nossa suposição estava errada, e na verdade 2 dos cadeados tem dígitos 
> do grupo (4, 5, 6, 7), basta fazer as mesmas tentativas, mas trocando os 
> dígitos (0, 1, 2, 3) por (4, 5, 6, 7). Assim, abriremos o armário em no 
> máximo 32 tentativas.
> 
> Em 26 de dezembro de 2016 08:26, Pedro José  escreveu:
>> Bom dia!
>> 
>> Fui inocente, fiz uma restrição que não precisava. Não há necessidade 
>> de acaso.
>> Pode haver estratégia.
>> 
>> Saudações,
>> PJMS
>> 
>> Em 25 de dezembro de 2016 12:31, Matheus Herculano 
>>  escreveu:
>>> 87
>>> 
>>> Em 23 de dez de 2016 13:07, "Gabriel Tostes"  escreveu:
>>>> Um armario de segurança tem 3 cadeados. Cada cadeado tem 8 combinacoes 
>>>> diferentes. O armario abre se quaisquer 2 dos 3 cadeados estao na posicao 
>>>> correta, qual e o numero minimo de tentativas pra abrir o armario?
>>>> 
>>>> 
>>>> --
>>>> Esta mensagem foi verificada pelo sistema de antivírus e
>>>> Â acredita-se estar livre de perigo.
>>>> 
>>>> 
>>>> =
>>>> Instruções para entrar na lista, sair da lista e usar a lista em
>>>> http://www.mat.puc-rio.br/~obmlistas/obm-l.html
>>>> =
>>> 
>>> -- 
>>> Esta mensagem foi verificada pelo sistema de antivírus e 
>>> acredita-se estar livre de perigo.
>> 
>> 
>> -- 
>> Esta mensagem foi verificada pelo sistema de antivírus e 
>> acredita-se estar livre de perigo.
> 
> 
> -- 
> Esta mensagem foi verificada pelo sistema de antivírus e 
> acredita-se estar livre de perigo.

-- 
Esta mensagem foi verificada pelo sistema de antiv�rus e
 acredita-se estar livre de perigo.



Re: [obm-l] Combinatoria

2016-12-23 Por tôpico Gabriel Tostes
24 nao eh possivel... 

> On Dec 23, 2016, at 16:22, Ralph Teixeira  wrote:
> 
> Hm, acho que dah para fazer com menos tentativas.
> 
> Sejam a, b e c as combinacoes corretas de cada cadeado, onde a,b,c
> estao em {0,1,2,3,4,5,6,7}.
> 
> Tentanto, por exemplo, todas as combinacoes possiveis para a e b
> (mantenha c=0), fazemos 64 tentativas, e com certeza vamos acabar
> acertando a combinacao dos dois primeiros cadeados -- o que eh
> suficiente para abrir o armario!
> 
> Mas eu nao estou dizendo que a resposta eh 64 -- acho que dah para ser
> mais esperto e abrir o armario garantidamente com menos tentativas...
> 
> (24, talvez?)
> 
> Abraco, Ralph.
> 
> P.S.: Pode me chamar de maluco, mas eu estou enxergando um cubo
> dividido em 8x8x8 cubinhos de LED, e a combinacao correta eh um
> cubinho especial desconhecido. Os 512 cubinhos comecam apagados; cada
> vez que voce faz uma tentativa, voce estah escolhendo um cubinho, e
> acendendo nao soh ele, mas todos os cubinhos na mesma linha, coluna
> ou... huh, outra linha. Em outras palavras, se voce escolhe o cubinho
> (A,B,C) (eu imagino voce botando o dedo nele para acende-lo, como se
> fosse uma jogada de um joguinho), voce acende todos os 22 cubinhos da
> forma (A,B,x), (A,x,C) ou (x,B,C) onde 0<=x<=7. Digo isso porque, se a
> combinacao correta dos 3 cadeados fosse um dos que acendeu, voce teria
> acertado pelo menos 2 cadeados, e assim abria o armario; e vice-versa,
> voce soh acerta se o cubinho desconhecido estiver entre esses 22.
> 
> Entao o problema eh o seguinte: qual a maneira mais economica (menos
> jogadas) de acender todos os 512 cubinhos no meu joguinho de LEDs? Eh,
> vai ter que acender **todos**, porque se voce esquecer unzinho, podia
> dar azar e ser aquela a combinacao correta, e entao voce nao garante
> abrir o armario!
> 
> Obviamente, como cada jogada acende 22, e sao 512 cubinhos, vamos
> precisar de no minimo 512/22, huh, arredonda para cima, 24 jogadas.
> Mas dah para fazer com 24? Para tanto, voce teria que ter muito poucas
> intersecoes entre jogadas distintas -- eh possivel?
> 
> 2016-12-23 14:53 GMT-02:00 Pedro José :
>> Bom dia!
>> 
>> Novamente o problema está mal formulado.
>> Embora possa parecer claro, qual é o número mínimo  de tentativas que
>> garanta abrir o armário.
>> 
>> Dois casos disjuntos atendem.
>> 
>> (i) Dois cadeados corretos e o outro errado.
>> 
>> Há uma chance de cada cadeado estar correto e 7 chances do terceiro estar
>> errado. Há 3 = C(3,2) jeitos de distribuir os dois cadeados corretos e o
>> errado.
>> 
>> Pelo princípio da multiplicação são: 3*7 = 21 eventos.
>> 
>> (ii) os três cadeados estão corretos;
>> 
>> Só há uma possibilidade.
>> 
>> O total de possibilidades para estar correto são 22 eventos.
>> 
>> O universo tem 8^3, logo há 8^3 -22 possibilidades que não abrem o 
>> armário.
>> 
>> Portanto para garantir que abra teremos 8^3 -22 +1 = 8^3 -21 = 491
>> tentativas.
>> 
>> Mas do jeito que o problema está formulado é 1. Se a pessoa der sorte de
>> acertar de primeira.
>> 
>> Saudações,
>> PJMS
>> 
>> 
>> 
>> 
>> 
>> Em 23 de dezembro de 2016 11:53, Gabriel Tostes 
>> escreveu:
>>> 
>>> Um armario de segurança tem 3 cadeados. Cada cadeado tem 8 combinacoes
>>> diferentes. O armario abre se quaisquer 2 dos 3 cadeados estao na posicao
>>> correta, qual e o numero minimo de tentativas pra abrir o armario?
>>> 
>>> 
>>> --
>>> Esta mensagem foi verificada pelo sistema de antivírus e
>>> acredita-se estar livre de perigo.
>>> 
>>> 
>>> =
>>> Instruções para entrar na lista, sair da lista e usar a lista em
>>> http://www.mat.puc-rio.br/~obmlistas/obm-l.html
>>> =
>> 
>> 
>> 
>> --
>> Esta mensagem foi verificada pelo sistema de antivírus e
>> acredita-se estar livre de perigo.
> 
> -- 
> Esta mensagem foi verificada pelo sistema de antivírus e
> acredita-se estar livre de perigo.
> 
> 
> =
> Instruções para entrar na lista, sair da lista e usar a lista em
> http://www.mat.puc-rio.br/~obmlistas/obm-l.html
> =

-- 
Esta mensagem foi verificada pelo sistema de antiv�rus e
 acredita-se estar livre de perigo.


=
Instru��es para entrar na lista, sair da lista e usar a lista em
http://www.mat.puc-rio.br/~obmlistas/obm-l.html
=


Re: [obm-l] Combinatoria

2016-12-23 Por tôpico Gabriel Tostes
Ola. O enunciado fala que se tiver no minimo dois exatos ele abre. Mas 491 nao 
e a resposta. Rapidamente e possível achar um limite bem inferior de 64 (mantem 
um constante e os outros dois usa as 64 possibilidades, uma delas deve abrir o 
armario). Na verdade a resposta e 32.

> On Dec 23, 2016, at 14:53, Pedro José  wrote:
> 
> Bom dia!
> 
> Novamente o problema está mal formulado.
> Embora possa parecer claro, qual é o número mínimo  de tentativas que 
> garanta abrir o armário.
> 
> Dois casos disjuntos atendem.
> 
> (i) Dois cadeados corretos e o outro errado.
> 
> Há uma chance de cada cadeado estar correto e 7 chances do terceiro estar 
> errado. Há 3 = C(3,2) jeitos de distribuir os dois cadeados corretos e o 
> errado.u
> 
> Pelo princípio da multiplicação são: 3*7 = 21 eventos.
> 
> (ii) os três cadeados estão corretos;
> 
> Só há uma possibilidade.
> 
> O total de possibilidades para estar correto são 22 eventos.
> 
> O universo tem 8^3, logo há 8^3 -22 possibilidades que não abrem o armário.
> 
> Portanto para garantir que abra teremos 8^3 -22 +1 = 8^3 -21 = 491 tentativas.
> 
> Mas do jeito que o problema está formulado é 1. Se a pessoa der sorte de 
> acertar de primeira.
> 
> Saudações,
> PJMS
> 
> 
> 
> 
> 
> Em 23 de dezembro de 2016 11:53, Gabriel Tostes  escreveu:
>> Um armario de segurança tem 3 cadeados. Cada cadeado tem 8 combinacoes 
>> diferentes. O armario abre se quaisquer 2 dos 3 cadeados estao na posicao 
>> correta, qual e o numero minimo de tentativas pra abrir o armario?
>> 
>> 
>> --
>> Esta mensagem foi verificada pelo sistema de antivírus e
>> Â acredita-se estar livre de perigo.
>> 
>> 
>> =
>> Instruções para entrar na lista, sair da lista e usar a lista em
>> http://www.mat.puc-rio.br/~obmlistas/obm-l.html
>> =
> 
> 
> -- 
> Esta mensagem foi verificada pelo sistema de antivírus e 
> acredita-se estar livre de perigo.

-- 
Esta mensagem foi verificada pelo sistema de antiv�rus e
 acredita-se estar livre de perigo.



[obm-l] Combinatoria

2016-12-23 Por tôpico Gabriel Tostes
Um armario de segurança tem 3 cadeados. Cada cadeado tem 8 combinacoes 
diferentes. O armario abre se quaisquer 2 dos 3 cadeados estao na posicao 
correta, qual e o numero minimo de tentativas pra abrir o armario?


-- 
Esta mensagem foi verificada pelo sistema de antiv�rus e
 acredita-se estar livre de perigo.


=
Instru��es para entrar na lista, sair da lista e usar a lista em
http://www.mat.puc-rio.br/~obmlistas/obm-l.html
=


[obm-l] Teoria dos numeros

2016-12-20 Por tôpico Gabriel Tostes
A,b,c,X,y,z inteiros tais que
a) ax^2+by^2+cz^2=abc +2xyz - 1
B) ab+bc+ca>=x^2+y^2+z^2

Provar que a,b,c são somas de 3 quadrados de inteiros



-- 
Esta mensagem foi verificada pelo sistema de antiv�rus e
 acredita-se estar livre de perigo.


=
Instru��es para entrar na lista, sair da lista e usar a lista em
http://www.mat.puc-rio.br/~obmlistas/obm-l.html
=


[obm-l] Algebra

2016-12-19 Por tôpico Gabriel Tostes
Alguem pode me explicar essa nota do mavropnevma no post #3 desse topico no 
aops?
http://www.artofproblemsolving.com/community/c6h461255p2587368

Ele escreveu z^6 -z^5+z^4-z^3+z^2-z+1 = b^2 como (16b)^2 = (16z^3-8z^2+6z-5)^2 
+140z^2-196z+231 e mostrou uma maneira de achar o polinomio dentro do ^2 de uma 
maneira rapida pela formula de Newton generalizada, mas eu n entendi.



-- 
Esta mensagem foi verificada pelo sistema de antivírus e
 acredita-se estar livre de perigo.


=
Instruções para entrar na lista, sair da lista e usar a lista em
http://www.mat.puc-rio.br/~obmlistas/obm-l.html
=


Re: [obm-l] Teorema de Eudoxius

2016-10-24 Por tôpico Gabriel Lopes
Talvez pelo Principio da boa ordenação rola

Em 24/10/2016 18:07, "Pedro Chaves"  escreveu:

> Caros Colegas,
>
> Como demonstrar, sem recorrer ao algoritmo da divisão euclidiana, o
> 'Teorema de Eudoxius':
>
> Dados os inteiros a e b, com b diferente de zero, então a é múltiplo de b
> ou se encontra entre dois múltiplos consecutivos de b.
>
> Obrigado a todos!
> Pedro Chaves
>
> --
> Esta mensagem foi verificada pelo sistema de antivírus e
> acredita-se estar livre de perigo.
>

-- 
Esta mensagem foi verificada pelo sistema de antiv�rus e
 acredita-se estar livre de perigo.



Re: [obm-l] Teoria dos Números

2016-09-26 Por tôpico Gabriel Tostes
O numero formado vai ser congruente a soma da soma dos algarismos desses dois 
numeros mod 3. Mas 2^n= (-1)^n e 2^n+1 = (-1)^n+1 somando os dois da sempre 0. 
Pois n+1 e n tem paridades diferentes. 

Sent from my iPad

> On Sep 26, 2016, at 16:37, Ricardo Leão  wrote:
> 
> Seja n um inteiro não negativo. Prove que o número formado colocando 2^n e 
> 2^(n+1) lado a lado em qualquer ordem é um múltiplo de 3.
> 
> Eu tentei resolver usando congruência, mas eu travei nessa questão.
> 
> Por favor, algum colega poderia fazer a demonstração?
> 
> -- 
> Esta mensagem foi verificada pelo sistema de antivírus e 
> acredita-se estar livre de perigo.

-- 
Esta mensagem foi verificada pelo sistema de antiv�rus e
 acredita-se estar livre de perigo.


=
Instru��es para entrar na lista, sair da lista e usar a lista em
http://www.mat.puc-rio.br/~obmlistas/obm-l.html
=


Re: [obm-l] Ajuda em Aritmética

2016-09-26 Por tôpico Gabriel Tostes
Um deles ser multiplo de 5 é equivalents a p^2 ser congruente a 1 ou p^2 ser 
congruente a 4, que são os unicos resíduos mod 5 além do 0. Logo P deve ser 
múltiplo de 5 e só testar P=5.

> On Sep 26, 2016, at 06:09, Marcelo de Moura Costa  wrote:
> 
> Bom dia a todos, um anulo me apresentou esse problema e confesso que pela 
> dica não consegui interpretá-lo corretamente e fiquei muito curioso como o 
> mesmo, será que alguém poderia me ajudar?
> O problema é:
> Mostre que somente para p=5, os números p, 4p^2+1 e 6p^2+1 serão primos. 
> (Dica: analise os restos da divisão de p por 5) 
> 
> Agradeço a atenção.
> 
> 
> -- 
> Esta mensagem foi verificada pelo sistema de antivírus e 
> acredita-se estar livre de perigo.

-- 
Esta mensagem foi verificada pelo sistema de antiv�rus e
 acredita-se estar livre de perigo.


=
Instru��es para entrar na lista, sair da lista e usar a lista em
http://www.mat.puc-rio.br/~obmlistas/obm-l.html
=


Re: [obm-l] Re: [obm-l] Análise combinatória

2015-12-10 Por tôpico Gabriel Tostes
Sim... Dividi em casos pra "tirar" a permutacao circular. O 136 de cada caso 
significa 136 modos de organizar as Cadeiras em "vazias" e "com Pessoas". Temos 
5! Maneiras de distribuir as Pessoas nelas.
> On Dec 10, 2015, at 17:34, Vanderlei Nemitz  wrote:
> 
> Gabriel:
> É justamente esse último 5! que eu tenho dúvidas. A permutação é 
> circular, certo? Mesmo assim multiplicamos por 5!? Sim, percebi o erro de 
> digitação, mas isso não é o principal. 
> 
> Em 10 de dezembro de 2015 17:23, Gabriel Tostes  escreveu:
>> A respostas 45360 está correta... Numere as cadeiras de 1 a 15 e dívida em 
>> 3 em casos:Â 
>> 1-> 15 ocupada
>> 2-> 1 ocupada (análogo ao 1º)
>> 3-> 1 e 15 vazias.
>> 
>> No primeiro caso temos que 1 e 14 devem estar vazias, logo, temos 4 pessoas 
>> para distribuir nas 12 cadeiras restantes... 
>> Como cada pessoa deve ocupar uma dessas cadeiras restam 8 vazias pra 
>> distribuir entre as pessoas, mas entre duas pessoas deve ter ao menos 
>> cadeira vazia, então -> 9!/5!x4!=136
>> No terceiro caso temos 13 cadeiras pra colocar 5 pessoas, logo 8 vazias, mas 
>> entre duas pessoas devemos ter uma cadeira vazia pelo menos uma vazia -> 
>> 9!/4!x5!=136
>> Total-> (2x136+136)x5!=45360
>> 
>>> On Dec 10, 2015, at 16:45, Vanderlei Nemitz  wrote:
>>> 
>>> Pessoal, gostaria de uma ajuda com essa questão. Vi em um site a 
>>> resposta 45360, mas não concordo. Encontrei um valor bem menor. Obrigado!
>>> 
>>> Vanderlei
>>> 
>>> Cinco pessoas devem se sentar em 15 cadeiras colocadas em torno de uma mesa 
>>> circular. De quantos modos isso pode ser feito se não deve haver 
>>> ocupação simultânea de duas cadeiras adjacentes? 
>>> 
>>> -- 
>>> Esta mensagem foi verificada pelo sistema de antivírus e 
>>> acredita-se estar livre de perigo.
>> 
>> -- 
>> Esta mensagem foi verificada pelo sistema de antivírus e 
>> acredita-se estar livre de perigo.
> 
> 
> -- 
> Esta mensagem foi verificada pelo sistema de antivírus e 
> acredita-se estar livre de perigo.

-- 
Esta mensagem foi verificada pelo sistema de antiv�rus e
 acredita-se estar livre de perigo.



Re: [obm-l] Análise combinatória

2015-12-10 Por tôpico Gabriel Tostes
9!/5!x4!=126, errei ali.

> On Dec 10, 2015, at 17:23, Gabriel Tostes  wrote:
> 
> A respostas 45360 está correta... Numere as cadeiras de 1 a 15 e dívida em 3 
> em casos: 
> 1-> 15 ocupada
> 2-> 1 ocupada (análogo ao 1º)
> 3-> 1 e 15 vazias.
> 
> No primeiro caso temos que 1 e 14 devem estar vazias, logo, temos 4 pessoas 
> para distribuir nas 12 cadeiras restantes... 
> Como cada pessoa deve ocupar uma dessas cadeiras restam 8 vazias pra 
> distribuir entre as pessoas, mas entre duas pessoas deve ter ao menos cadeira 
> vazia, então -> 9!/5!x4!=136
> No terceiro caso temos 13 cadeiras pra colocar 5 pessoas, logo 8 vazias, mas 
> entre duas pessoas devemos ter uma cadeira vazia pelo menos uma vazia -> 
> 9!/4!x5!=136
> Total-> (2x136+136)x5!=45360
> 
>> On Dec 10, 2015, at 16:45, Vanderlei Nemitz  wrote:
>> 
>> Pessoal, gostaria de uma ajuda com essa questão. Vi em um site a resposta 
>> 45360, mas não concordo. Encontrei um valor bem menor. Obrigado!
>> 
>> Vanderlei
>> 
>> Cinco pessoas devem se sentar em 15 cadeiras colocadas em torno de uma mesa 
>> circular. De quantos modos isso pode ser feito se não deve haver ocupação 
>> simultânea de duas cadeiras adjacentes? 
>> 
>> -- 
>> Esta mensagem foi verificada pelo sistema de antivírus e 
>> acredita-se estar livre de perigo.

-- 
Esta mensagem foi verificada pelo sistema de antiv�rus e
 acredita-se estar livre de perigo.



Re: [obm-l] Análise combinatória

2015-12-10 Por tôpico Gabriel Tostes
A respostas 45360 está correta... Numere as cadeiras de 1 a 15 e dívida em 3 em 
casos: 
1-> 15 ocupada
2-> 1 ocupada (análogo ao 1º)
3-> 1 e 15 vazias.

No primeiro caso temos que 1 e 14 devem estar vazias, logo, temos 4 pessoas 
para distribuir nas 12 cadeiras restantes... 
Como cada pessoa deve ocupar uma dessas cadeiras restam 8 vazias pra distribuir 
entre as pessoas, mas entre duas pessoas deve ter ao menos cadeira vazia, então 
-> 9!/5!x4!=136
No terceiro caso temos 13 cadeiras pra colocar 5 pessoas, logo 8 vazias, mas 
entre duas pessoas devemos ter uma cadeira vazia pelo menos uma vazia -> 
9!/4!x5!=136
Total-> (2x136+136)x5!=45360

> On Dec 10, 2015, at 16:45, Vanderlei Nemitz  wrote:
> 
> Pessoal, gostaria de uma ajuda com essa questão. Vi em um site a resposta 
> 45360, mas não concordo. Encontrei um valor bem menor. Obrigado!
> 
> Vanderlei
> 
> Cinco pessoas devem se sentar em 15 cadeiras colocadas em torno de uma mesa 
> circular. De quantos modos isso pode ser feito se não deve haver ocupação 
> simultânea de duas cadeiras adjacentes? 
> 
> -- 
> Esta mensagem foi verificada pelo sistema de antivírus e 
> acredita-se estar livre de perigo.

-- 
Esta mensagem foi verificada pelo sistema de antiv�rus e
 acredita-se estar livre de perigo.



Re: [obm-l] Probabilidade em urnas

2015-12-09 Por tôpico Gabriel Tostes
Nao entendi muito bem se eh exatamente 2 ou 2 ou 3. Se for exatamente 2->
Devemos tirar 3 pretas e 2 vermelhas e temos 10 ordens possiveis para fazer 
isso. A probabilidade de qualquer ordem dessa ocorrer eh 6x5x4x3x2/9x8x7x6x5. A 
probabilidade eh 10 vezes a probabilidade de uma ordem certa de tirada das 
bolas ocorrer. 10/21.
Se entrar as possibilidades com 3 bolas fica:
10/21+10x6x5x3x2x1/9x8x7x6x5 = 25/42

Sent from my iPad

> On Dec 10, 2015, at 00:18, João Sousa  wrote:
> 
> Caso, em uma urna, sejam colocadas 6 bolas pretas e 3 bolas vermelhas e 
> decida-se retirar dessa urna , sem reposição, 5 bolas, guardando-se em um 
> recipiente a parte, qual a probabilidade de, nesse recipiente, haver 2 bolas 
> vermelhas?
> 
> João Sousa
> 
> -- 
> Esta mensagem foi verificada pelo sistema de antivírus e 
> acredita-se estar livre de perigo.

-- 
Esta mensagem foi verificada pelo sistema de antiv�rus e
 acredita-se estar livre de perigo.



Re: [obm-l] Função Convexidade

2015-12-07 Por tôpico Gabriel Tostes
Segunda derivada eh -senx , vai ser negativo pra qualquer valor entre 0 e pi






Sent from my iPad
> On Dec 7, 2015, at 09:42, Israel Meireles Chrisostomo 
>  wrote:
> 
> Olá rapazes, será que alguém poderia confirmar para mim que a função 
> √senx é côncova no intervalo (0,pi/2)? 
> 
> -- 
> Esta mensagem foi verificada pelo sistema de antivírus e 
> acredita-se estar livre de perigo.

-- 
Esta mensagem foi verificada pelo sistema de antiv�rus e
 acredita-se estar livre de perigo.



Re: [obm-l] Re: [obm-l] equação diofantina

2015-10-14 Por tôpico Gabriel Tostes
(1,0) nao eh solucao tbm?



Sent from my iPad
> On Oct 14, 2015, at 11:04, Israel Meireles Chrisostomo 
>  wrote:
> 
> Está aqui no site do professor Diego Marques: 
> http://diego.mat.unb.br/click.html
> Só possui uma solução, que é a solução trivial(x=3 e y=2).Mas  o 
> difícil é provar que a solução é única, veja que raciocínio 
> fantástico!
> 
> Em 14 de outubro de 2015 07:41, marcone augusto araújo borges 
>  escreveu:
>> E a solução da equação 3^x - 5^y = 2 ?
>> 
>> -- 
>> Esta mensagem foi verificada pelo sistema de antivírus e 
>> acredita-se estar livre de perigo.
> 
> 
> -- 
> Esta mensagem foi verificada pelo sistema de antivírus e 
> acredita-se estar livre de perigo.

-- 
Esta mensagem foi verificada pelo sistema de antiv�rus e
 acredita-se estar livre de perigo.



Re: [obm-l] Ajuda

2015-10-14 Por tôpico Gabriel Tostes
Pra N tem raizes reais a^2 - 4a^2 + 24 < 0 a>2sqrt2
Podemos admitir a real, caso contrario, a equacao obviamente nao possui raízes 
reais.
Devemos provar que nao existe raiz de a menor que 2sqrt2
Se f(X)=x^3-6x-6
Como f(2sqrt2).f(-oo)>0 f(X) tem um numero par de raizes entre ]-oo,2sqrt2]
Ou seja, 0 ou 2 solucoes.
Agora, como 
f(2sqrt2)f(2.03sqrt2)<0 temos uma ou 3 solucoes nesse intervalo. Obviamente 
temos uma solucao visto que a soma das solucoes e igual a 0.
Chamando essa solucao de x3
X1+x2=-x3
X1.x2=6/x3
Entao para x1 e x2 nao serem reais temos que (x3)^2 -24/x3 < 0 => x3<24^(1/3) 
de fato, pois x3 esta entre 2Sqrt2 e 2.03sqrt2. Temos que x3 é a unica soluçao 
real da equacao e eh maior que 2sqrt2.




Sent from my iPad
> On Oct 14, 2015, at 07:57, marcone augusto araújo borges 
>  wrote:
> 
> Seja a um número real tal que a^3 = 6(a+1).Mostre que a equação
> x^2 + ax+ a^2 - 6 = 0 não tem raízes reais.
> 
> -- 
> Esta mensagem foi verificada pelo sistema de antivírus e 
> acredita-se estar livre de perigo.

-- 
Esta mensagem foi verificada pelo sistema de antiv�rus e
 acredita-se estar livre de perigo.



Re: [obm-l] Equação diofantina

2015-10-13 Por tôpico Gabriel Tostes
Usando : pros tres pauzinhos da congruencias.

3^x=2 + 5^y
3^x:2 (mod5)
X=4K+3
3^(4k+3)=2+5^y
5^y:7(mod9)
y=6k+2
5^6k+2:25:4(mod7)
3^x:2+4(mod7)


> On Oct 13, 2015, at 22:00, Israel Meireles Chrisostomo 
>  wrote:
> 
> Não quero que resolvam a equação pois já tenho a solução, só quero 
> entender uma parte da solução...Na equação 3^x-5^y=2, como posso concluir 
> que 3^x é congruente 6 módulo 7?Alguém poderia me explicar como concluir 
> isso?
> 
> -- 
> Esta mensagem foi verificada pelo sistema de antivírus e 
> acredita-se estar livre de perigo.

-- 
Esta mensagem foi verificada pelo sistema de antiv�rus e
 acredita-se estar livre de perigo.


=
Instru��es para entrar na lista, sair da lista e usar a lista em
http://www.mat.puc-rio.br/~obmlistas/obm-l.html
=


[obm-l] Generalizando um questão de combinatória

2015-10-13 Por tôpico gabriel araujo guedes
 No livro de combinatória do Morgado seção de permutação caótica, questão
8, diz:
"Dois médicos devem examinar, durante uma mesma hora, 6 pacientes, gastando
10 minutos com cada paciente.Cada um dos 6 pacientes deve ser examinado
pelos dois médicos. De quanto modos pode ser feito um horário compatível?"

A resposta obtida  é 6!*D_6, no qual D_6 é a permutação caótica de 6
elementos.

A questão pode ser facilmente para n pacientes, obtendo n!*D_n como
resposta.

Agora se considerarmos k médicos e n pacientes, com k

[obm-l] Problema 6 da OBM de 2002

2015-10-11 Por tôpico Gabriel Tostes
Mostre que não podemos formar mais que 4096 sequências binárias de tamanho 24 
tal que quaisquer 2 diferem em ao menos 8 posições.
Não consegui entender a resolução na Eureka. Alguém pode resolvê-lo?


Sent from my iPad
-- 
Esta mensagem foi verificada pelo sistema de antiv�rus e
 acredita-se estar livre de perigo.


=
Instru��es para entrar na lista, sair da lista e usar a lista em
http://www.mat.puc-rio.br/~obmlistas/obm-l.html
=


[obm-l] Funcao Injetora

2015-09-03 Por tôpico Gabriel Tostes
f(x) + f(f(x)) = 2x implica que f(x) é injetora? Porque? 
Domínio e contra dominio são os reais não negativos sem o zero.
-- 
Esta mensagem foi verificada pelo sistema de antiv�rus e
 acredita-se estar livre de perigo.


=
Instru��es para entrar na lista, sair da lista e usar a lista em
http://www.mat.puc-rio.br/~obmlistas/obm-l.html
=


[obm-l] Polinômios

2015-07-08 Por tôpico Gabriel Tostes
Ache o resto de x^100 -2.x^51 + 1 na divisao por x^2 - 1.
Eu nao entendo por que o resto eh 4x nao  -2x + 2 
Se fizer x=1 nao fica a + b = 0 ? E x=-1 -a+b=4 r(x) = ax + b
Esse exercicio ta no livro do Engel, problem solving strategies.


-- 
Esta mensagem foi verificada pelo sistema de antivírus e
 acredita-se estar livre de perigo.

=
Instruções para entrar na lista, sair da lista e usar a lista em
http://www.mat.puc-rio.br/~obmlistas/obm-l.html
=


Re: [obm-l] Re: [obm-l] Re: Irredutibilidade de polinômios

2015-05-25 Por tôpico Gabriel Tostes
Vlw! Realmente nao tinha nada a ver pensar desse jeito... Resolvi de outro 
jeito aqui... Quando x for 0 esse polinomio tem que ser múltiplo de 9, mas ele 
e igual a 3. 

Enviada do meu iPad

> Em 25/05/2015, às 09:30, Bernardo Freitas Paulo da Costa 
>  escreveu:
> 
> 2015-05-24 21:51 GMT-03:00 Gabriel Tostes :
>> Esta certo eu provar isso dizendo que, pelo teorema da raiz racional, as 
>> unicas solucoes inteiras podem ser -1, 1, 3 e -3 mas que, com essa opcoes, 
>> tal polinomio nunca sera igual a 0?
> Não. Pegue dois polinômios irredutíveis em Z[x] sem raízes racionais.
> Tipo P = x^2 + 1 e Q = x^2 + 2. O produto deles também não tem raízes
> racionais, mas é redutível.
> 
> Abraços,
> -- 
> Bernardo Freitas Paulo da Costa
> 
> -- 
> Esta mensagem foi verificada pelo sistema de antivírus e
> acredita-se estar livre de perigo.
> 
> 
> =
> Instruções para entrar na lista, sair da lista e usar a lista em
> http://www.mat.puc-rio.br/~obmlistas/obm-l.html
> =

-- 
Esta mensagem foi verificada pelo sistema de antiv�rus e
 acredita-se estar livre de perigo.


=
Instru��es para entrar na lista, sair da lista e usar a lista em
http://www.mat.puc-rio.br/~obmlistas/obm-l.html
=


[obm-l] Re: Irredutibilidade de polinômios

2015-05-24 Por tôpico Gabriel Tostes
Esta certo eu provar isso dizendo que, pelo teorema da raiz racional, as unicas 
solucoes inteiras podem ser -1, 1, 3 e -3 mas que, com essa opcoes, tal 
polinomio nunca sera igual a 0?

> Em 24/05/2015, às 21:39, Gabriel Tostes  escreveu:
> 
> (IMO) Prove que o polinomio x^n + 5x^(n-1) + 3 é irredutivel em Z[x]
> 
> Alguma ideia pra essa questão?

-- 
Esta mensagem foi verificada pelo sistema de antiv�rus e
 acredita-se estar livre de perigo.


=
Instru��es para entrar na lista, sair da lista e usar a lista em
http://www.mat.puc-rio.br/~obmlistas/obm-l.html
=


Re: [obm-l] Re: [obm-l] Combinatória

2015-05-24 Por tôpico Gabriel Tostes
Se você for escolhendo todos os números, irá ter 9 opções para o primeiro, 10 
pra o segundo, terceiro,,oitavo. Mas somente terá 5 opções para o último 
número. 

Enviada do meu iPad

> Em 24/05/2015, às 15:38, Rogerio Ponce  escreveu:
> 
> A sequencia comeca com um IMPAR e a segunda e' PAR, e vao se alternando 
> sucessivamente...
> 
> 2015-05-24 15:35 GMT-03:00 Rogerio Ponce :
>> Oi Bernardo, obrigado, engoli "a soma".
>> Indo de um em um, a "soma" do primeiro e' par, a proxima e' impar, etc.
>> (afinal o Marcone nao queria saber quantos numeros pares existiam na 
>> sequencia...)
>> :)
>> []'s
>> Rogerio Ponce
>> 
>> 2015-05-24 12:56 GMT-03:00 Rogerio Ponce :
>>> Ola' Marcone,
>>> os numeros de 9 algarismos comecam em 1, e terminam em 9.
>>> Indo de um em um, o primeiro e' par, o proximo e' impar, o seguinte e' par, 
>>> etc...
>>> A sequencia comeca com um par e termina com um impar.
>>> Portanto tem a mesma quantidade de elementos pares e impares.
>>> Ou seja, 45000 elementos pares e 45000 elementos impares.
>>> []'s
>>> Rogerio Ponce
>>> 
>>> 2015-05-23 21:31 GMT-03:00 marcone augusto araújo borges 
>>> :
 Quantos números de 9 algarismos tem a soma dos seus algarismos par?
 
 Eu achei 45000.Não tenho o gabarito.
 Notei que esse número é a metade do total de números de 9 algarismos
 Seria metade dos números com soma dos seus algarismos par e metade
 com soma dos algarismos ímpar.Se isso for verdade, é mera coincidência
 ou teria como justificar?
 
 -- 
 Esta mensagem foi verificada pelo sistema de antivírus e 
 acredita-se estar livre de perigo.
> 
> 
> -- 
> Esta mensagem foi verificada pelo sistema de antivírus e 
> acredita-se estar livre de perigo.

-- 
Esta mensagem foi verificada pelo sistema de antiv�rus e
 acredita-se estar livre de perigo.



[obm-l] Irredutibilidade de polinômios

2015-05-24 Por tôpico Gabriel Tostes
(IMO) Prove que o polinomio x^n + 5x^(n-1) + 3 é irredutivel em Z[x]

Alguma ideia pra essa questão?
-- 
Esta mensagem foi verificada pelo sistema de antiv�rus e
 acredita-se estar livre de perigo.


=
Instru��es para entrar na lista, sair da lista e usar a lista em
http://www.mat.puc-rio.br/~obmlistas/obm-l.html
=


Re: [obm-l] Re: [obm-l] Como faz exercícios desse tipo?

2015-05-10 Por tôpico Gabriel Tostes
Obrigado! Eu n conheço mto bem essa de razões da unidade, pode me indicar algum 
pdf
que explica isso?



> Em 10/05/2015, às 10:53, Jeferson Almir  escreveu:
> 
> OBs: w^k= cis(2kPi/6)
> 
> Em domingo, 10 de maio de 2015, Jeferson Almir  
> escreveu:
>> Raízes da unidade!! ... Pelo algoritmo da divisão temos g(x^12) = g(x)q(x) 
>> + r(x) , onde grau(r(x)) <5 agora vc analisa as raízes da unidade de x^6=1 
>> : que serão w^k=1 onde k=0,1,2,3,4,5 e monta o sistema sobre r(x) aplicando 
>> o valor dessas raízes pois r(x) = ax^4 + bx^3 + cx^2 + dx + e  elas irão 
>> zerar g(x) agora é resolver o sistema utilizando as propriedades das 
>> raízes da unidade. 
>> 
>> Em domingo, 10 de maio de 2015, Israel Meireles Chrisostomo 
>>  escreveu:
>>> Talvez vc poderia observar que -1 é raiz do polinômio,daí vc pode  
>>> fatorar o polinômio como (x-(-1))Q(x) e talvez procurar outras raízes, pq 
>>> aí vc pode fazer a divisão por binômios do tipo (x+1) pois assim  vc 
>>> resolve facilmente pelo algoritmo de briott ruffini, conhece?
>>> 
>>> Em 9 de maio de 2015 18:42, Gabriel Tostes  escreveu:
>>>> (EUA/83) Sabendo que g(x) = x^5 + x^4 + x^3 + x^2 + x + 1. Calcule o resto 
>>>> da divisão entre polinômios g(x^12) e g(x) 
>>>> 
>>>> Dado f(x) =  x^4 + x^3 + x^2 + x + 1, o resto da divisão de f(x^5) por 
>>>> f(x) é: 
>>>> 
>>>> -- 
>>>> Esta mensagem foi verificada pelo sistema de antivírus e 
>>>> acredita-se estar livre de perigo.
>>> 
>>> 
>>> -- 
>>> Esta mensagem foi verificada pelo sistema de antivírus e 
>>> acredita-se estar livre de perigo.
> 
> -- 
> Esta mensagem foi verificada pelo sistema de antivírus e 
> acredita-se estar livre de perigo.

-- 
Esta mensagem foi verificada pelo sistema de antiv�rus e
 acredita-se estar livre de perigo.



[obm-l] Como faz exercícios desse tipo?

2015-05-09 Por tôpico Gabriel Tostes
(EUA/83) Sabendo que g(x) = x^5 + x^4 + x^3 + x^2 + x + 1. Calcule o resto da 
divisão entre polinômios g(x^12) e g(x) 

Dado f(x) =  x^4 + x^3 + x^2 + x + 1, o resto da divisão de f(x^5) por f(x) é: 
-- 
Esta mensagem foi verificada pelo sistema de antiv�rus e
 acredita-se estar livre de perigo.



[no subject]

2015-03-29 Por tôpico Gabriel Tostes
Alguém me ajuda a responder? 
determine as raízes reais da equação:
X^4 + 16x - 12 = 0
-- 
Esta mensagem foi verificada pelo sistema de antiv�rus e
 acredita-se estar livre de perigo.


=
Instru��es para entrar na lista, sair da lista e usar a lista em
http://www.mat.puc-rio.br/~obmlistas/obm-l.html
=


[obm-l] Re: [obm-l] Re: [obm-l] Romênia

2015-02-28 Por tôpico Gabriel Lopes
Obrigado.

Em 27 de fevereiro de 2015 12:55, Esdras Muniz 
escreveu:

> Bem, para a bijeção só falta mostrar a injeção, suponha por absurdo x f(x)=f(y), a sequência x, y, x, y, x, y,  é divergente, mas sua imagem
> não, pois é constante, já q f(x)=f(y).
> Agora, suponha a inversa "g" descontínua, então existe e>0, e x real tais
> que para todo n natural,
> |g(x)-g(y)|>e, para |x-y|<1/n. Então vc faz x=f(a) e y=f(bn), onde a
> sequência bn é divergente, assim fica:
> |a-bn|>e (já que bn diverge) além disso |f(a)-f(bn)|<1/n, o que implica
> que f(bn) converge para f(a), gerando um absurdo.
> Talvez haja algum erro bobo que precise ser corrigido, mas acho q é isso.
>
> Em 27 de fevereiro de 2015 08:37, Gabriel Lopes 
> escreveu:
>
>> *Gostaria de ajuda com a seguinte questão vinda da Romênia , acho que da
>> olimpíada (o livro não especifica qual olímpíada e qual ano) :
>>
>> - Seja f: R --> R  uma função sobrejetiva , satisfazendo a seguinte
>> propriedade : para toda sequência divergente (a_n) , n > = 1 ,  a sequência
>> (f(a_n)) , n> = 1 , também é divergente .  Prove que  f  é bijetiva e que
>> sua função inversa f^(-1) é contínua.
>>
>> *O livro oferta a seguintes dicas :
>>
>> 1.(Para provar que f é bijetiva) Tome x,y  em R distintos e considere
>> (a_n) ,n > = 1 , a sequência divergente tal que a_2k = x  e  a_2k-1 = y ,
>> para todo k > = 1 , e utilize a segunda hipótese do enunciado .
>>
>> * Aqui deduzimos que existe e > 0  tal que  para todo n* em N  temos: m,n
>> > n*,  m > n ,  então   | f(a_m) - f(a_n) | = | f(x) - f(y) | > =  e  ;
>> pela relação entre sequências convergentes e  sequências de Cauchy ,  e
>> então negando a afirmação :  ( f(a_n) ) ,n > =1 ,  é convergente .
>>
>> 2.(Para provar que f^(-1)  é contínua) Use as Hipóteses sobre f  para
>> mostrar que f^(-1) transforma sequências convergentes em sequências
>> convergentes.
>>
>> *Parei por aqui mas os seguintes comentários são pertinentes :
>>
>> I.  O  capítulo do livro em que tirei este problema fala sobre
>> Continuidade Sequencial  e prova  o seguinte TMA : Uma função f: I --> R
>> , onde I é um intervalo, é contínua se , e só se , a seguinte condição for
>> satisfeita : para todo a  em  I   e  toda sequência (a_n),n > = 1, de
>> elementos de I ,  temos : lim(a_n)  = a   , então  lim( f(a_n) ) =  f(a)
>> .  (não consegui só com ele)
>>
>> II. Procurei sobre o TMA em outro livro ,Curso de Análise Vol.1  Elon
>> Lages , Capítulo VII , e encontrei o seguinte corolário :  A fim de que f
>> seja contínua  no ponto  a  , é suficiente que , para toda sequência  de
>> pontos a_n  de  X   ( creio  que  X  é uma união de Intervalos)   com
>> lim( a_n )  =  a   ,   exista   lim( f(a_n) ) .  O mesmo não foi
>> demonstrado ,e  também não consegui faze-lo  , mas acho que ele é
>> suficiente para resolver a questão.
>>
>>
>> --
>> Esta mensagem foi verificada pelo sistema de antivírus e
>> acredita-se estar livre de perigo.
>
>
>
>
> --
> Esdras Muniz Mota
> Mestrando em Matemática
> Universidade Federal do Ceará
>
>
>
> --
> Esta mensagem foi verificada pelo sistema de antivírus e
> acredita-se estar livre de perigo.

-- 
Esta mensagem foi verificada pelo sistema de antiv�rus e
 acredita-se estar livre de perigo.



[obm-l] Romênia

2015-02-27 Por tôpico Gabriel Lopes
*Gostaria de ajuda com a seguinte questão vinda da Romênia , acho que da
olimpíada (o livro não especifica qual olímpíada e qual ano) :

- Seja f: R --> R  uma função sobrejetiva , satisfazendo a seguinte
propriedade : para toda sequência divergente (a_n) , n > = 1 ,  a sequência
(f(a_n)) , n> = 1 , também é divergente .  Prove que  f  é bijetiva e que
sua função inversa f^(-1) é contínua.

*O livro oferta a seguintes dicas :

1.(Para provar que f é bijetiva) Tome x,y  em R distintos e considere (a_n)
,n > = 1 , a sequência divergente tal que a_2k = x  e  a_2k-1 = y , para
todo k > = 1 , e utilize a segunda hipótese do enunciado .

* Aqui deduzimos que existe e > 0  tal que  para todo n* em N  temos: m,n >
n*,  m > n ,  então   | f(a_m) - f(a_n) | = | f(x) - f(y) | > =  e  ; pela
relação entre sequências convergentes e  sequências de Cauchy ,  e então
negando a afirmação :  ( f(a_n) ) ,n > =1 ,  é convergente .

2.(Para provar que f^(-1)  é contínua) Use as Hipóteses sobre f  para
mostrar que f^(-1) transforma sequências convergentes em sequências
convergentes.

*Parei por aqui mas os seguintes comentários são pertinentes :

I.  O  capítulo do livro em que tirei este problema fala sobre Continuidade
Sequencial  e prova  o seguinte TMA : Uma função f: I --> R , onde I é um
intervalo, é contínua se , e só se , a seguinte condição for satisfeita :
para todo a  em  I   e  toda sequência (a_n),n > = 1, de elementos de I ,
temos : lim(a_n)  = a   , então  lim( f(a_n) ) =  f(a) .  (não consegui só
com ele)

II. Procurei sobre o TMA em outro livro ,Curso de Análise Vol.1  Elon Lages
, Capítulo VII , e encontrei o seguinte corolário :  A fim de que f seja
contínua  no ponto  a  , é suficiente que , para toda sequência  de pontos
a_n  de  X   ( creio  que  X  é uma união de Intervalos)   com  lim( a_n )
=  a   ,   exista   lim( f(a_n) ) .  O mesmo não foi demonstrado ,e  também
não consegui faze-lo  , mas acho que ele é suficiente para resolver a
questão.

-- 
Esta mensagem foi verificada pelo sistema de antiv�rus e
 acredita-se estar livre de perigo.



[obm-l] Re: [obm-l] Re: [obm-l] Equação funcional e Continuidade

2015-02-24 Por tôpico Gabriel Lopes
Aparentemente o caso de f  decrescente não era análogo , Obrigado Ralph.

Em 22 de fevereiro de 2015 22:19, Ralph Teixeira 
escreveu:

> Tem funcoes demais... Basicamente:
>
> i) Escolha um a qualquer tal que 0 ii) Desenhe um grafico continuo decrescente QUALQUER de (0,1) ateh (a,a).
> iii) Desenhe o simetrico deste grafico com relacao aa reta y=x
> iv) Pronto, voce tem um grafico de funcao que satisfaz suas condicoes!
>
> Abraco, Ralph.
>
> 2015-02-20 14:36 GMT-05:00 Gabriel Lopes :
>
>> *Prezados colegas gostaria de ajuda com o seguinte problema:
>>
>> - Encontre todas as funções contínuas  f : [0,1] --> [0,1]  tais que:
>> f(f(x)) = x  .
>>
>> *Procedi da seguinte maneira:
>>
>> 1.Deduzi imediatamente (pelos fatos básicos de composição de funções)
>> que  f  é bijetiva .
>>
>> 2.Na continuação utilizei do seguinte TMA :  Se  f : X --> R  é uma
>> função contínua  , então f é injetiva  se e somente se é crescente ou
>> decrescente.
>>
>> 3.Não consegui ir alem , olhei então a dica do meu livro que procedeu
>> como eu fiz em 1 e 2 , e acresceu o seguinte : I. Suponha que  f  é
>> crescente ( o caso em que f  é decrescente é análogo) , II. Suponha que
>> para algum  x  em  (0,1)  :  f(x) > x   então  x = f(f(x)) > f(x)  ,uma
>> contradição e da mesma forma eliminamos o caso  f(x) < x  ;  portanto  f(x)
>> = x  , para todo x em [0,1] .
>>
>> 4.O problema fica quando tento provar o caso em que  f  é decrescente (
>> que parece não ser  completamente análogo) ; obviamente a função  f(x) = 1
>> - x   também satisfaz  , logo tentei obter uma contradição ao supor  f(x) <
>> 1 - x  para algum x em (0,1)  ; parei por aqui.
>>
>> *Sinto que talvez seja uma coisa boba ( alguma manipulação algébrica
>> simples etc...) contudo não consegui continuar ;  se  for algo mais
>> complexo poderiam enviar uma dica junto a solução?
>>
>> --
>> Esta mensagem foi verificada pelo sistema de antivírus e
>> acredita-se estar livre de perigo.
>
>
>
> --
> Esta mensagem foi verificada pelo sistema de antivírus e
> acredita-se estar livre de perigo.

-- 
Esta mensagem foi verificada pelo sistema de antiv�rus e
 acredita-se estar livre de perigo.



[obm-l] Equação funcional e Continuidade

2015-02-20 Por tôpico Gabriel Lopes
*Prezados colegas gostaria de ajuda com o seguinte problema:

- Encontre todas as funções contínuas  f : [0,1] --> [0,1]  tais que:
f(f(x)) = x  .

*Procedi da seguinte maneira:

1.Deduzi imediatamente (pelos fatos básicos de composição de funções) que
f  é bijetiva .

2.Na continuação utilizei do seguinte TMA :  Se  f : X --> R  é uma função
contínua  , então f é injetiva  se e somente se é crescente ou decrescente.

3.Não consegui ir alem , olhei então a dica do meu livro que procedeu como
eu fiz em 1 e 2 , e acresceu o seguinte : I. Suponha que  f  é crescente (
o caso em que f  é decrescente é análogo) , II. Suponha que para algum  x
em  (0,1)  :  f(x) > x   então  x = f(f(x)) > f(x)  ,uma contradição e da
mesma forma eliminamos o caso  f(x) < x  ;  portanto  f(x) = x  , para todo
x em [0,1] .

4.O problema fica quando tento provar o caso em que  f  é decrescente ( que
parece não ser  completamente análogo) ; obviamente a função  f(x) = 1 - x
  também satisfaz  , logo tentei obter uma contradição ao supor  f(x) < 1 -
x  para algum x em (0,1)  ; parei por aqui.

*Sinto que talvez seja uma coisa boba ( alguma manipulação algébrica
simples etc...) contudo não consegui continuar ;  se  for algo mais
complexo poderiam enviar uma dica junto a solução?

-- 
Esta mensagem foi verificada pelo sistema de antiv�rus e
 acredita-se estar livre de perigo.



[obm-l] Provas antigas OBM

2015-02-06 Por tôpico Gabriel Ayres do Nascimento
Olá!

Alguém tem as provas (completas) da OBM de 1995, 1996 e 1997 (17ª, 18ª e 19ª 
edições)?

Dessas, o site da OBM disponibiliza apenas a de 1997, mas só a prova do nível 
júnior (apesar de estar discriminado júnior e sênior).

Gabriel Ayres


-- 
Esta mensagem foi verificada pelo sistema de antivírus e
 acredita-se estar livre de perigo.


=
Instruções para entrar na lista, sair da lista e usar a lista em
http://www.mat.puc-rio.br/~obmlistas/obm-l.html
=


[obm-l] Re: [obm-l] Re: Re: Re: Re: Função

2014-09-23 Por tôpico Gabriel Lopes
Obrigado a ambos , acho que entendi o que faltou no meu raciocínio.

Em 21 de setembro de 2014 22:10, Listeiro 037 
escreveu:

>
> Foi esse o resultado que obtive.
>
> Nesse procedimento que fiz ocorre um cancelamento de zero com outro
> zero: 0/0.
>
> A minha dúvida é se esse expediente de tratar com matrizes 2x2 é válido
> (que funciona, funciona sim) e onde eu encontraria mais material sobre
> essa técnica.
>
>
> Em Sun, 21 Sep 2014 18:31:04 -0300
> saulo nilson  escreveu:
>
> > Seja  f: R --> R , uma função definida por :
> >(x+a)/(x+b) , sex  é diferente de  -b
> > f(x) =
> >-1   , se  x  é igual a  -b
> >
> > Se  f(f(x)) = x  , para todo x  real , encontre o valor de   ab .
> > f(1)=(a+1)/(1+b)
> > 1=((a+1)/(1+b)+a)/((a+1)/(b+1)+b)
> > 1=(a+1+a+ab)/(a+1+b^2+b)
> > -1=(a-1+ab-a)/(a-1+b^2-b)
> > 2+2b=2a+2ab
> > 1+b=a+ab
> > 0=(a+ab)/(a+b^2)
> > a(1+b)=0
> > a=0
> > b=-1
> >
> >
> >
> >
> >
> > 2014-09-19 0:04 GMT-03:00 Listeiro 037 :
> >
> > >
> > > Eu fui direto ao cálculo de f(f(x)) = x. Nisto
> > >
> > > (((x+a)/(x+b))+a)/(((x+a)/(x+b))+b)=x, substituições sucessivas.
> > >
> > > Fiz sem levar em conta o f(-b) = -1.
> > >
> > > Existe uma teoria que usa uma notação matricial em expressões do
> > > tipo (ax+b)/(cx+d), melhor (az+b)/(cz+d), que embora o contexto
> > > seja de números complexos, dá certo usar produtos sucessivos de
> > > matrizes no caso de em (ax+b)/(cx+d) querer substituir x por
> > > (a'x'+b')/(c'x'+d'). Com x diferente de (-d/c) e x' diferente de
> > > (-d'/c'), que é onde o denominador se anula.
> > >
> > > (ax+b)/(cx+d) em forma de matriz fica
> > >
> > > [a b]
> > > [c d]
> > >
> > >
> > > (x+a)/(x+b) em forma de matrix fica
> > >
> > > [1 a]
> > > [1 b]
> > >
> > > apenas x fica
> > >
> > > [1 0]
> > > [0 1]
> > >
> > > que é (x+0)/(0x+1)
> > >
> > > Neste caso, aqui no problema proposto encontrei a=0 e b=-1, sem
> > > considerar f(-b).
> > >
> > >
> > > Em Thu, 18 Sep 2014 12:58:20 -0300
> > > Gabriel Lopes  escreveu:
> > >
> > > > Fiquei sem entender sua explicação , poderia elaborar um pouco
> > > > mais?
> > > >
> > > > Pensei no seguinte:
> > > >
> > > >
> > > > Observe que  :
> > > >
> > > >(x+a)/(x+a)  = 1   , se  x é
> > > > diferente de  -b
> > > > a  =  b   ==>f (x) =
> > > >-1  , se
> > > > x é igual a  -b
> > > >
> > > >
> > > >
> > > > Temos então uma contradição pois : f(f(x)) = x . Donde  a  é
> > > > diferente de b .
> > > >
> > > >
> > > > Mas :
> > > >
> > > > f(f(-a)) = f(0) = -a  ,  (substituindo em : (x+a)/(x+b) ) .
> > > >
> > > > e:
> > > >
> > > > f(f(-b)) = f(-1) = -b
> > > >
> > > >
> > > > Donde:
> > > >
> > > > (a/b) = -a  , se0  é diferente de  -b
> > > > f(0) =
> > > >-1 = -a  , se   0 é igual a  -b .
> > > >
> > > >
> > > > Portanto  :
> > > >
> > > > f(f(-a)).f(f(-b)) = (-a).(-b) = ab  = f(0).(-b)  ,  donde:
> > > >
> > > >
> > > > ab = -a   , se  0 é diferente de  -b
> > > >
> > > > ab = a.0 = b = -1(-b) = 0 , se 0 é igual a  -b .
> > > >
> > > >
> > > > Em 18 de setembro de 2014 06:07, Listeiro 037
> > > >  escreveu:
> > > >
> > > > >
> > > > > A função aplicada à ela mesma. Pode ser feito assim?
> > > > > Produto de duas matrizes 2x2 igualado à matriz identidade 2x2?
> > > > >
> > > > > [1 a; 1 b] [1 a; 1 b] = [1 0; 0 1]
> > > > >
> > > > > [1 a] [1 a]  [1 0]
> > > > > [1 b] [1 b]  [0 1]
> > > > >
> > > > > [1+a a+ab; 1+b a+b^2]
> > > > >
> > > > > [1+a a+ab ]
> > > > > [1+b a+b^2]
> > > > >
> > > > &

[obm-l] Re: [obm-l] Re: Função

2014-09-18 Por tôpico Gabriel Lopes
Fiquei sem entender sua explicação , poderia elaborar um pouco mais?

Pensei no seguinte:


Observe que  :

   (x+a)/(x+a)  = 1   , se  x é diferente
de  -b
a  =  b   ==>f (x) =
   -1  , se  x é  igual
a  -b



Temos então uma contradição pois : f(f(x)) = x . Donde  a  é diferente de
b .


Mas :

f(f(-a)) = f(0) = -a  ,  (substituindo em : (x+a)/(x+b) ) .

e:

f(f(-b)) = f(-1) = -b


Donde:

(a/b) = -a  , se0  é diferente de  -b
f(0) =
   -1 = -a  , se   0 é igual a  -b .


Portanto  :

f(f(-a)).f(f(-b)) = (-a).(-b) = ab  = f(0).(-b)  ,  donde:


ab = -a   , se  0 é diferente de  -b

ab = a.0 = b = -1(-b) = 0 , se 0 é igual a  -b .


Em 18 de setembro de 2014 06:07, Listeiro 037 
escreveu:

>
> A função aplicada à ela mesma. Pode ser feito assim?
> Produto de duas matrizes 2x2 igualado à matriz identidade 2x2?
>
> [1 a; 1 b] [1 a; 1 b] = [1 0; 0 1]
>
> [1 a] [1 a]  [1 0]
> [1 b] [1 b]  [0 1]
>
> [1+a a+ab; 1+b a+b^2]
>
> [1+a a+ab ]
> [1+b a+b^2]
>
> Aparentemente a=0 e b=1.
>
>
> Em Wed, 17 Sep 2014 09:30:08 -0300
> Gabriel Lopes  escreveu:
>
> > Seja  f: R --> R , uma função definida por :
> >
> >
> >(x+a)/(x+b) , sex  é diferente de  -b
> > f(x) =
> >-1   , se  x  é igual a  -b
> >
> >
> >
> > Se  f(f(x)) = x  , para todo x  real , encontre o valor de   ab .
> >
>
>
> --
> Esta mensagem foi verificada pelo sistema de antivírus e
>  acredita-se estar livre de perigo.
>
>
> =
> Instru�ões para entrar na lista, sair da lista e usar a lista em
> http://www.mat.puc-rio.br/~obmlistas/obm-l.html
> =
>

-- 
Esta mensagem foi verificada pelo sistema de antiv�rus e
 acredita-se estar livre de perigo.



[obm-l] Função

2014-09-17 Por tôpico Gabriel Lopes
Seja  f: R --> R , uma função definida por :


   (x+a)/(x+b) , sex  é diferente de  -b
f(x) =
   -1   , se  x  é igual a  -b



Se  f(f(x)) = x  , para todo x  real , encontre o valor de   ab .

-- 
Esta mensagem foi verificada pelo sistema de antiv�rus e
 acredita-se estar livre de perigo.



Re: [obm-l] PBO

2014-09-14 Por tôpico Gabriel Haeser
Na verdade vc precisa usar o princípio da indução pra provar o PBO (é a
formalização da sua prova intuitiva). Suponha que A não tem um menor
elemento. Seja S o complementar de A. Temos que 0 não está em A, caso
contrário 0 seria o menor elemento de A. Então 0 está em S. Assuma que
0,1,...,k estão em S. Então k+1 está em S, senão, k+1 seria o menor
elemento de A. Pelo princípio da indução (forte), S=N, o que torna A vazio,
absurdo.

On Monday, September 15, 2014, Eduardo Henrique 
wrote:

> Fiz uma demonstraçãozinha do PBO, queria opiniões por favor.
>
> Teorema: Seja A \in N um conjunto não-vazio. A tem um menor elemento.
>
> Demonstração:
>
> Suponho que A seja um conjunto não-vazio de números naturais sem um menor
> elemento.
> Seja p_{0} \in A. Como A não possui menor elemento, temos que existe p_{1}
> menor que p_{0} em A. Raciocinando analogamente, temos uma cadeia infinita
> de números naturais menores do que p_{0} em A, mas isso é contradição pois
> o conjunto Nn={p \in N | 1 \leq p \leq p_{0}} é finito. Logo A é vazio
>
>
> --
> Esta mensagem foi verificada pelo sistema de antivírus e
> acredita-se estar livre de perigo.
>

-- 
Esta mensagem foi verificada pelo sistema de antiv�rus e
 acredita-se estar livre de perigo.



Re: [obm-l] Re: [obm-l] Re: [obm-l] Re: [obm-l] Divisíbilidade

2014-08-16 Por tôpico Gabriel Tostes
xyq=x^2+y^2+1 q=x/y + y/x +1/xy. Como q é inteiro positivo. 1/xy também. X=y=1 
q=1+1+1=3 alguém mandou essa? Acho q é o jeito mais fácil

Enviada do meu iPad

> Em 16/08/2014, às 16:22, Douglas Oliveira de Lima 
>  escreveu:
> 
> É verdade Bernardo Freitas , da pra ver que funciona com os números de 
> fibonacci, a saber:Â 
> (F2n-1)^2+(F2n+1)^2+1=3(F2n-1)(F2n+1), onde F2n-1 é um número de Fibonacci, 
> por exemplo quando n=3Â 
> teríamos (F5)^2+(F7)^2+1=3(F5)(F7), assim 1,1,2,3,5,8,13,21,34,..., 
> 8^2+21^2+1=3.8.21Â 
> ( Que legal!! como se prova isso?)
> 
> Douglas Oliveira
> Â Â 
> 
> 
> Em 15 de agosto de 2014 22:33, Bernardo Freitas Paulo da Costa 
>  escreveu:
>> 2014-08-15 22:01 GMT-03:00 Bernardo Freitas Paulo da Costa
>> :
>> > Eu acho que continua errado...
>> >
>> > 2014-08-15 11:20 GMT-03:00 Pedro José :
>> >> x, y Ɛ Z+  e  xy | x^2 + y^2 +1 ==> x | x^2 + y^2 +1 (i)
>> >> x | x^2 e (i) ==> x | y^2 + 1 (Combinação Z linear de x^2 + y^2 +1 e 
>> >> x^2)
>> >> ==> Ǝ k Ɛ Z | kx = y^2 + 1 (ii)
>> >> (ii) e por simetria da proposta ==>  Ǝ m Ɛ Z | my = x^2 + 1 ==> y =( 
>> >> x^2 +
>> >> 1)/m (iii)
>> >> (ii) e (iii) ==> kx = (x^4 + 2x^2 +2)/m^2 ==> m^2k x = x^4 + 2x^2 +2 (iv)
>> >
>> > (ii) kx = y^2 + 1
>> > (iii) y = (x^2 + 1)/m
>> >
>> > Donde y^2 = (x^4 + 2x^2 + 1)/m^2
>> > Donde kx = (x^4 + 2x^2 + 1)/m^2 + 1 = (x^4 + 2x^2 + (1 + m^2))/m^2
>> > (e não +2)
>> >
>> > O resto talvez funcione mais ou menos igual... mas dá mais trabalho...
>> >
>> >> m^2k Ɛ Z (v), pois +, * e potênciação são fechadas em Z.
>> >> (iv) e (v) ==> x | x^4 + 2x^2 +2 (vi)
>> >>
>> >> x | x^4 + 2x^2 (vii)
>> >> (vi) e (vii) ==>  x | 2 ( Z combinação linear de x^4 + 2x^2 e x^4 + 
>> >> 2x^2 +1)
>> >
>> > (k*m*m)*x = (x^4 + 2x^2 + (1 + m*m)) => x | 1 + m^2
>> >
>> > Com um pouco de trabalho, você acha também a solução x = 2, y = 5:
>> > 10 | 4 + 25 + 1 = 30. E o quociente continua igual a 3 (como
>> > gostaríamos de demonstrar...)
>> 
>> Bom, com um pouco mais de paciência... você acha a solução x = 5, y =
>> 13. E daí você chuta que a próxima solução é x = 13, y = 34, porque
>> números de Fibonacci são legais... e dá certo: 34*13 = 442, 13*13 +
>> 34*34 + 1 = 1326 = 3 * 442. Mágica?
>> 
>> Eu acho que há infinitas soluções. Deixo vocês provarem isso. Agora
>> resta ver que são apenas estas soluções!
>> 
>> Abraços,
>> --
>> Bernardo Freitas Paulo da Costa
>> 
>> --
>> Esta mensagem foi verificada pelo sistema de antivírus e
>> Â acredita-se estar livre de perigo.
>> 
>> 
>> =
>> Instru�ões para entrar na lista, sair da lista e usar a lista em
>> http://www.mat.puc-rio.br/~obmlistas/obm-l.html
>> =
> 
> 
> -- 
> Esta mensagem foi verificada pelo sistema de antivírus e 
> acredita-se estar livre de perigo.

-- 
Esta mensagem foi verificada pelo sistema de antiv�rus e
 acredita-se estar livre de perigo.



Re: [obm-l] Lista 4 Cone Sul 2008

2014-05-19 Por tôpico Gabriel Lopes
Estou sem ideias ...

No site  do Treinamento Cone Sul onde encontrei a questão , os
organizadores não disponibilizaram as resoluções das listas...

Alguém tem ideia de como resolver a questão?


Em 17 de maio de 2014 16:03, Vanderlei Nemitz escreveu:

> Saulo, não entendi. Para mostrar que a função é injetiva, uma maneira é
> mostrar que f(x1) = f(x2) implica em x1 = x2. Além disso, é n^ 2007 e não
> n!^2007. Concorda?
> Em 17/05/2014 15:36, "saulo nilson"  escreveu:
>
> n1!(n1!^2006-1)=f(n1)
>>
>> n2!(n2!^2006-1)=f(n2)
>> n1=n2
>> f(n1)=f(n2)
>> n1=!n2
>> f(n1)=!f(n2)
>>
>>
>>
>>
>> 2014-05-17 10:47 GMT-03:00 Gabriel Lopes :
>>
>>> 9 . Prove que a função f : N --> Z definida por :
>>>
>>> f(n) = (n^2007) − n!
>>>
>>> é injetiva.
>>>
>>> --
>>> Esta mensagem foi verificada pelo sistema de antivírus e
>>> acredita-se estar livre de perigo.
>>
>>
>>
>> --
>> Esta mensagem foi verificada pelo sistema de antivírus e
>> acredita-se estar livre de perigo.
>
>
> --
> Esta mensagem foi verificada pelo sistema de antivírus e
> acredita-se estar livre de perigo.
>

-- 
Esta mensagem foi verificada pelo sistema de antiv�rus e
 acredita-se estar livre de perigo.



Re: [obm-l] Lista 4 Cone Sul 2008

2014-05-17 Por tôpico Gabriel Lopes
Poderia elaborar mais um pouco ? Não compreendi as passagens.

Obs: Talvez eu que não tenha entendido mas , no enunciado consta como :

f(n) : ( n elevado a 2007) menos ( n fatorial)  ;  e não :

f(n) : ( n fatorial elevado a 2007) menos ( n fatorial)


Em 17 de maio de 2014 15:32, saulo nilson  escreveu:

> n1!(n1!^2006-1)=f(n1)
>
> n2!(n2!^2006-1)=f(n2)
> n1=n2
> f(n1)=f(n2)
> n1=!n2
> f(n1)=!f(n2)
>
>
>
>
> 2014-05-17 10:47 GMT-03:00 Gabriel Lopes :
>
>> 9 . Prove que a função f : N --> Z definida por :
>>
>> f(n) = (n^2007) − n!
>>
>> é injetiva.
>>
>> --
>> Esta mensagem foi verificada pelo sistema de antivírus e
>> acredita-se estar livre de perigo.
>
>
>
> --
> Esta mensagem foi verificada pelo sistema de antivírus e
> acredita-se estar livre de perigo.

-- 
Esta mensagem foi verificada pelo sistema de antiv�rus e
 acredita-se estar livre de perigo.



[obm-l] Lista 4 Cone Sul 2008

2014-05-17 Por tôpico Gabriel Lopes
9 . Prove que a função f : N --> Z definida por :

f(n) = (n^2007) − n!

é injetiva.

-- 
Esta mensagem foi verificada pelo sistema de antiv�rus e
 acredita-se estar livre de perigo.



Re: [obm-l] Somatório

2014-04-26 Por tôpico Gabriel Tostes
Primeiro você toma 3 somas: 1 - 1 + 1 - 1 ... = s1 1-2+3-4+5-6+... = s2 
1+2+3+4+5...=s3

A primeira vai dar 1/2 pois se parar em um número ímpar dá 1 e se parar em um 
par da 0. A segunda se você somá-la a ela mesma mas com um zero na frente 
(1-2+3-4+5-6+...) + (0+1-2+3-4+5-6+...) vai dar 1-1+1-1... = s1 = 1/2 = 2s2, 
então s2 = 1/4... Por fim: se subtrair s3 de s2 dará o somatório de todos 
múltiplos de 4 -> 4(1+2+3+4...) = s3 -s2 -> 4(s3) = s3 - 1/4 -> s3 = -1/12 que 
é o somatório de todos naturais.

> Em 12/04/2014, às 12:53, Vanderlei Nemitz  escreveu:
> 
> Pessoal, vi em um site a seguinte camiseta:
> 
> http://www.zazzle.com.br/teoria_da_corda-235032240070858893
> 
> Lembrei que uma vez um aluno meu disse que tinha visto uma "prova" de que a 
> soma dos infinitos números naturais era negativa. Não consegui encontrar na 
> época e agora vi outra vez vez na camiseta. Alguém sabe como explicar esse 
> absurdo ou então existe alguma explicação física, como diz o site?
> 
> Obrigado!
> 
> 
> -- 
> Esta mensagem foi verificada pelo sistema de antivírus e 
> acredita-se estar livre de perigo

-- 
Esta mensagem foi verificada pelo sistema de antiv�rus e
 acredita-se estar livre de perigo.



[obm-l] (Torneio das Cidades) Polinômios

2014-04-12 Por tôpico Gabriel Lopez
 Olá pessoal ,  tenho uma dúvida quanto a resolução  da seguinte  questão
que encontrei no material de Álgebra Nível 3 do POTI (aula 06)   :

(Torneio das Cidades)


Sabendo que a equação :
   x^4 + ax³ + 2x²
+ bx + 1 = 0

possui uma raiz real, prove que :

   a² + b²  *>
ou =*  8


Solução . Temos que:

   x^4 + ax³ + 2x³ + bx + 1 = (x² + px + q)(x²
+ sx + t)   (1)

em que *p, q, s, t* são *reais*. Como, pelo menos uma das raízes são reais,
iremos
assumir que ela é raiz de x²+ sx + t, então:

  s² > ou = 
4t.

Igualando os coeficientes em (1), temos:

 a = p + s

 2 = q + t + ps;

 b = pt + qs;

 1 = qt.


Portanto,

a² + b² = p² + q² + 2ps + p²t² + q²s² + 2ptqs = p²(1 + t²) + s²(1 + q²) +
4ps *> ou =* p²(1 + t²) + 4(t + q + ps) *> ou =*  8.



Minha dúvida é a seguinte:

Como foi feita a dedução que os os coeficientes a e b são reais? Eles não
poderiam pertencer a : C -IR ?

Para mim  a solução faria sentido se no enunciado constasse que  a e b são
reais , ou quem sabe se fosse :


   | a² + b² |  *> ou =  *8 .



Grato.

-- 
Esta mensagem foi verificada pelo sistema de antivírus e
 acredita-se estar livre de perigo.



[obm-l] Composição de Funções periódicas

2014-01-10 Por tôpico Gabriel Ayres do Nascimento

Fala pessoal,

Seja f uma função periódica de R em R e g uma função qualquer de R em R. A 
função composta gof é necessariamente periódica? E a função fog? Demonstre, 
caso afirmativo, ou dê um contra exemplo, cado contrário.

Deem uma ideia aí.

Gabriel Ayres

-- 
Esta mensagem foi verificada pelo sistema de antivírus e
 acredita-se estar livre de perigo.


=
Instruções para entrar na lista, sair da lista e usar a lista em
http://www.mat.puc-rio.br/~obmlistas/obm-l.html
=


[obm-l] Re: [obm-l] Cálculo

2013-12-25 Por tôpico Gabriel Haeser
Defina h=f-g e use o teorema do valor intermediario.

On Wednesday, December 25, 2013, Vanderlei Nemitz wrote:

> Alguém poderia me ajudar na seguinte questão? Muito obrigado e um feliz
> Natal!
>
> *Sejam f e g funções contínuas num intervalo [a, b], tais que f(a) < g(a)
> e f(b) > g(b). Prove que existe um número c entre a e b, tal que f(c) =
> g(c).*
>
> --
> Esta mensagem foi verificada pelo sistema de antivírus e
> acredita-se estar livre de perigo.

-- 
Esta mensagem foi verificada pelo sistema de antivírus e
 acredita-se estar livre de perigo.



Re: [obm-l] Apostila de Desenho 2 Impacto OFF TOPIC

2013-12-10 Por tôpico Gabriel Ayres do Nascimento
Olá!

Também estou interessado por tal material de desenho geométrico. Como faço para 
obtê-lo?

Desde já agradeço!

Gabriel Nascimento


Em ter, 10/12/13, Jeferson Almir  escreveu:

 Assunto: Re: [obm-l] Apostila de Desenho 2 Impacto OFF TOPIC
 Para: "obm-l@mat.puc-rio.br" 
 Data: Terça-feira, 10 de Dezembro de 2013, 17:09
 
 Oi João trabalho nos
 dois programas do governo referente a olimpiada de
 matemática o POT e OBMEP aqui em fortaleza do meu belo
 ceará  e há tempos procuro um material de desenho
 geométrico para trabalhar com os meninos pois sinto a
 carência deles em problemas que requer construçoes
 geométricas e reforço o meu interesse sobre o tão
 recomendado material. Jeferson Almir
 
 
 Em
 terça-feira, 10 de dezembro de 2013, Graciliano Antonio
 Damazo escreveu:
 
 Boa tarde, acabei de receber minha
 apostila.
 Obrigado.
 Graciliano
 
  
   
Em Terça-feira, 10 de
 Dezembro de 2013 10:51, Mauricio de Araujo 
 escreveu:
 
 Apostila
 recebida, muito obrigado!!!
 
 2013/12/4 Jefferson
 Franca 
 
 
 
 
 
 
 Muito obrigado 
  
 
 
  
 
Em Terça-feira, 3 de
 Dezembro de 2013 17:36, "jjun...@fazenda.ms.gov.br" 
 escreveu:
 
 
 
 Prezados
 amigos, 
 Não há
 necessidade de reembolso. Os valores gastos não
 foram elevados. Vocês serão úteis a vários e
 distintos jovens pelo país, e,
  assim, eu também, indiretamente...
 Fraternalmente, João.
 
 
 
 - Mensagem
 Original -
 
 
 
 De: obm-l@mat.puc-rio.br
 Para:"obm-l@mat.puc-rio.br" 
 
 
 
 Cópia:~e
 Enviado:Tue, 3 Dec 2013
 11:57:08 -0800 (PST)
 Assunto:Re: [obm-l] Apostila
 de Desenho 2 Impacto OFF TOPIC
 
 
 
 
 
 Obrigado João pelo envio do
 material.
 
 
 Me informe o numero e agencia de sua conta para que eu possa
 lhe pagar os custos de xerox e correio (e mais tempo perdido
 para fazer o serviço).
 
 
 Mais uma vez agradeço e o que precisar é só
 avisar.
 
 
 AbraçosGraciliano
 
 
  
 
  
 
 
Em Terça-feira, 3 de
 Dezembro de 2013 15:43, Carlos Victor  escreveu:

 
 Obrigado João,
 Envie-me a sua conta bancária para
 depósito, ok ?
 Também posso lhe enviar o custo
 pelo correio.
 
 
 
 Agradeço e fico a disposição para o que
 precisares.
 
 Abraços
 Carlos Victor
 
 Em 1 de dezembro de 2013
 17:54,  
 escreveu:
 
 
 
  Senhores: 
 
 Ontem (sábado), por volta das 15h em Campo Grande, foi
 enviada cópia da Apostila 2 de Desenho do IMPACTO, uma ao
 senhor Carlos Victor (Nilópolis - RJ), e outra a Graciliano
 Antônio Damazo (Penápolis - SP).
 
 
 
 
 ATT.João (Campo Grande - MS)
 
 

 --
 
 Esta mensagem foi verificada pelo sistema
 de antivírus e 
 
  acredita-se estar livre de perigo.
 
 
 
 --
 
 Esta mensagem foi verificada pelo sistema
 de antivírus e 
 
  acredita-se estar livre de perigo.


-- 
Esta mensagem foi verificada pelo sistema de antivírus e
 acredita-se estar livre de perigo.


=
Instruções para entrar na lista, sair da lista e usar a lista em
http://www.mat.puc-rio.br/~obmlistas/obm-l.html
=


[obm-l] Generalizaçao do Principio de Cavalieri

2013-08-27 Por tôpico Gabriel Ayres do Nascimento
Saudaçoes!

O seguinte problema foi tirado do livro Espaço e Forma do glorioso Elon Lages:

"Se todo plano horizontal pi secciona os solidos A e B segundo figuras planas 
pi_inter_A e pi_inter_B tais que a area pi_inter_A eh k vezes a area de 
pi_inter_B, onde k eh uma constante, entao vol(A)=k.vol(B)."

Alguma sugestao?

Gabriel Ayres
-- 
Esta mensagem foi verificada pelo sistema de antivírus e
 acredita-se estar livre de perigo.



[obm-l] Uma resposta

2013-05-14 Por tôpico Gabriel Lisboa
Aos amantes da Matemática, Teoria dos Números em particular:

Provaram uma das versões da Conjectura de Goldbach... Chorei!

https://plus.google.com/u/0/114134834346472219368/posts/8qpSYNZFbzC


[obm-l] RE: [obm-l] Re: [obm-l] Equações(inteiros)

2012-12-03 Por tôpico Gabriel Franco

como parar de receber esses emails? 
  

[obm-l] Re: [obm-l] ajuda em exercício de desigualdade

2012-12-01 Por tôpico Gabriel Dalalio
Tira raiz de 100 dos dois lados:

n^2 < 5^3 = 125 , o maior quadrado perfeito que não passa de 125 é 121,
maior valor para n é 11

Gabriel Dalalio

Em 2 de dezembro de 2012 00:01, Bruno Rodrigues
escreveu:

>  Olá galera,estou travado nesse problema que segue:
>
> Ache o maior valor inteiro positivo de n tal que:
>  n^²°°<5^³°°
>
> alguém poderia dar uma luz?
> abraços
> Bruno
>


[obm-l] Re: [obm-l] Princípio de Dir... Casa dos Pombos

2012-10-28 Por tôpico Gabriel Dalalio
Dá pra provar que existe um retangulo num grid de tamanho 9 x 3 pintado de
vermelho e azul.

Você só tem 8 possibilidades de pintar uma linha de 3 pontos com duas
cores, então pegando 9 linhas você terá duas linhas iguais. Nessas linhas
iguais vai aparecer um retangulo de 4 vertices de cor igual, por que numa
linha de tres pontos uma das das cores aparece em pelo menos dois pontos, e
como as duas linhas são iguais você consegue formar um retângulo de uma cor
só.

Em 28 de outubro de 2012 22:19, Athos Couto  escreveu:

>  Em um plano, são pintados de azul ou vermelho, de maneira aleatória,
> todos os pontos de coordenadas inteiras.
> Prove que existe no mínimo um retângulo nesse plano cujos vértices são de
> uma mesma cor.
>


[obm-l] Re: [obm-l] Re: [obm-l] Problema sobre existência de subconjunto divisível

2012-10-15 Por tôpico Gabriel Dalalio
Eu pensei em casa dos pombos mas não consegui muita coisa, arranjar um
subconjunto qualquer que a soma seja divisível por n é facil, o problema é
ter exatamente n elementos.

Em 15 de outubro de 2012 20:24, terence thirteen
escreveu:

> Em 15 de outubro de 2012 18:49, Gabriel Dalalio
>  escreveu:
> > Eae galera, beleza?
> >
> > Eu estou pensando na seguinte situação:
> >
> > É dado um conjunto de inteiros de 2n elementos.
> > Sempre existe um subconjunto de n elementos tal que sua soma é divisível
> por
> > n?
>
> Talvez um casa-dos-pombos?
>
> > E será que sempre existem pelo menos dois subconjuntos diferentes com
> essa
> > propriedade?
> >
> > Eu só consegui achar exemplos com no mínimo dois subconjuntos possíveis,
> > por exemplo, um conjunto formado por n elementos 0 e n elementos 1.
> >
> > Alguém sabe responder essas perguntas?
> >
> > Obrigado,
> > Gabriel Dalalio
>
>
>
> --
> /**/
> 神が祝福
>
> Torres
>
> =
> Instru�ões para entrar na lista, sair da lista e usar a lista em
> http://www.mat.puc-rio.br/~obmlistas/obm-l.html
> =
>


[obm-l] Re: [obm-l] Abaixar o nível da aula

2012-06-03 Por tôpico Gabriel Merêncio
Posso falar apenas como aluno, mas espero que seja relevante à
discussão. Acredito que a escola deva ser um agente auxiliar à formação do
indivíduo, possibilitando um desenvolvimento pleno e sadio. Desse ponto de
vista, é muito bom que você queira oferecer algo além que pode complementar
a bagagem de conhecimento do aluno, mas, ao mesmo tempo, não dá para querer
impor a todos.

Não vejo como questão de abaixar o nível, porém adequar-se ao contexto: não
são todos que verão o conteúdo como algo significativo em suas vidas. Uma
boa parte só tem interesse em matemática até onde o vestibular cobra, o que
é perfeitamente compreensível. Aliás, o que parece trivial pode ser um
verdadeiro pesadelo aos que, por exemplo, preferem dedicar-se ao estudo de
idiomas ou textos filosóficos de pensadores.

Uma boa alternativa são aulas extras fora do horário normal voltadas aos
alunos interessados; por exemplo, muitas escolas têm cursos preparatórios
para olimpíadas.

2012/6/2 Marco Antonio Leal 

>  Sou professor de matemática em Belém do Pará e sempre tento incentivar os
> alunos a estudar forte, buscar mais problemas, falo e resolvo problemas
> sobre olimpíadas, mostro teoremas como menelaus, ceva e demonstro todos os
> teoremas, mas, para minha surpresa, os alunos se preocupam apenas em tentar
> resolver problemas triviais das universidades estadual, federal e Cesupa,
> que é uma universidade particular. Estas universidades junto com o ENEM
> cobram problemas triviais, sem profundidade e imediatos que, na minha
> opinião, não selecionam os melhores candidatos nem fazem jus ao conteudo
> ministrado. Me deixa muito triste esse fato, ja que, começo a perceber que
> uma geração de alunos esta se formando, onde o contexto da questão é mais
> importante do que o conteudo. Gostaria de saber dos meus colegas de
> profissão se passam pela mesma angustia em suas escolas, melhor ainda, se
> para ser um bom professor, é necessario baixar o nível da aula e excluir a
> abordagem mais profunda do conteudo
>


Re: [obm-l] Fibonacci

2012-04-08 Por tôpico Gabriel Guedes
Oi amigos da lista.
Bernardo, mas ai estaria implícito nas suas hipóteses que a quantidade dos
que morrem é igual as do que nasceram a certo tempo atrás. Acredito que
deveriam existir três relações F para os nascimentos ( que é a seq de
Fibonacci que conhecemos). G uma outra para a morte dos coelhos. E uma H em
função de F e G para modelar o novo problema.
O que acha?

Em 8 de abril de 2012 03:49, Bernardo Freitas Paulo da Costa <
bernardo...@gmail.com> escreveu:

> 2012/4/8 Rogerio Ponce :
> > Ola'  Gabriel,
> > se cada casal viver por k+0.5 meses (0.5 e' para nao haver confusao
> > sobre a geracao de descendentes no momento em que o casal morre),
> > entao basta voce subtrair a quantidade de coelhos com idade igual ou
> > mais velhos que k+1 meses.
> > Assim, a resposta para o seu problema seria
> > F(n) - F(n-k-1)
> Bom, vou dizer que eu achei estranha essa resposta porque a sua
> seqüência satisfaz a mesma recorrência que o problema original,
> enquanto que eu acho que a recorrência aqui é
>
> G(n+k+2) = G(n+k+1) + G(n+k) - G(n)
>
> (Ou seja, entre o mês n+k+1 e o seguinte, os coelhos que têm mais de 1
> mês geram um novo casal, o que corresponde ao termo "+ G(n+k)", e os
> que são bem velhinhos morrem, o que dá o termo "- G(n)" ; isso dá
> k+1+0.5 meses de vida para os coelhos, seguindo a sua idéia).
>
> Ora,
>
> F(n+k+2) - F(n+alfa+2) = F(n+k+1) + F(n+k) - (F(n+alfa+1) + F(n+alfa))
> = [F(n+k+1) - F(n+alfa+1)] + [F(n+k) - F(n+alfa)]
>
> que é a recorrência de Fibonacci (claro) para X(n) = F(n+k) -
> F(n+alfa). A mesma demonstração diz que nenhuma combinação de F(n)'s
> pode ser solução da recorrência modificada.
>
> Daí eu acho que você esqueceu de subtrair também os filhos que F(n)
> inclui para os coelhos "velhos demais".
>
> Quanto à recorrência que eu propus acima, eu acho que ela é bem mais
> chata de resolver porque o polinômio característico depende de k;
> x^(k+2) = x^(k+1) + x^k - 1.
>
> Sem computador, você já pode perceber que há sempre uma solução x = 1.
> Se k for ímpar, você também tem x = -1. Também sem computador, você
> pode acreditar que a maior solução é sempre menor do que Phi = (1 +
> raiz(5))/2, porque tem que haver menos coelhos do que no caso que eles
> são imortais, e você pode até chutar que a maior solução é
> "crescente". Com um computador, você pode continuar chutando: as
> raízes reais são apenas as que eu mostrei acima, e as outras são
> complexas de módulo menor do que 1. Talvez dê pra provar isso sem
> muito trabalho, mas sei lá. Eu acho que a gente também pode chutar que
> o módulo delas é maior do que o módulo de phi = (1 - raiz(5))/2, mas
> eu não tenho grandes justificativas pra isso não. Também parece que o
> módulo delas tende a 1 (ou seja de todas as raízes exceto a maior), e
> talvez isso seja mais fácli de demonstrar.
>
> Ah, outra coisa importante de chutar (depois disso tudo) é que as
> raízes são todas simples, porque daí basta saber qual é a maior, e o
> coeficiente das raízes 1 e -1 para achar os valores para n grande por
> truncamento ;)
>
> Abraços,
> --
> Bernardo Freitas Paulo da Costa
>
> =
> Instruções para entrar na lista, sair da lista e usar a lista em
> http://www.mat.puc-rio.br/~obmlistas/obm-l.html
> =
>


[obm-l] Fibonacci

2012-04-07 Por tôpico Gabriel Guedes
Caros colegas da lista,
alguem conhece um texto sobre o problema dos coelhos  de Fibonacci,
mas que troque a hipótese dos coelhos nunca morrerem, por uma hipótese
dos coelhos morrerem após um determinado período de tempo?
Atenciosamente,
Gabriel Guedes


[obm-l] valor de aderencia

2012-04-02 Por tôpico Luan Gabriel
Galera,se alguém puder ajudar nessa:
V ou F : o conjunto dos valores de aderencia de uma sequência eh sempre 
enumeravel.

justificar.

vlw
luan gabriel
  

[obm-l] RE: [obm-l] RE: [obm-l] TEORIA DOS NÚMEROS

2012-03-24 Por tôpico Gabriel Franco


como faço para não receber mais esses emails ?
From: joao_maldona...@hotmail.com
To: obm-l@mat.puc-rio.br
Subject: [obm-l] RE: [obm-l] TEORIA DOS NÚMEROS 
Date: Sat, 24 Mar 2012 13:45:20 -0300







Eu elaborei  uma solução que diria "FEIA", na verdade uma bonita seria uma 
fatoração que em aparecesse facilmente o 97
Enfim, fatorando o 111Chamando y de 333^555 + 555^333y = 111^333(3^333 333^222 
+ 5^333) = 111^333 ((3^5 111^2)^111 + (5^3)^111)Mas 
(3^5 111^2)^111  = (243*14*14)^111 = 1 (mod 97)5^3 = 28(5^3)^111 = (28^3)^37 = 
-1 mod 97
Logo y é divisível por 97
[]'sJoão

Date: Sat, 24 Mar 2012 07:45:16 -0300
Subject: [obm-l] TEORIA DOS NÚMEROS
From: vanderma...@gmail.com
To: obm-l@mat.puc-rio.br

Pessoal, vejam a seguinte questão:
 
Prove que 333^555 + 555^333 é múltiplo de 97.
 
Tentei de tudo, mas não consegui.
 
Um abraço,
 
Vanderlei


  

[obm-l] Re: [obm-l] Dúvidas

2012-03-15 Por tôpico Gabriel Merêncio
1 - João atravessa o percurso BT, de 12 km, com velocidade média de 15
km/h, o que significa que ele leva 12/15 * 60 = 48 minutos. Daí, restam 42
minutos para a trajetória CB, a qual João percorre com velocidade média de
10 km/h. Em 42 minutos (ou seja, 42/60 = 7/10 de hora), João percorre uma
distância de 7 km, que é o valor de CB.

Agora note que o ângulo CBA é suplementar ao ângulo CBT, então os senos são
equivalentes. Como o seno é cateto oposto sobre hipotenusa: 0,54 = AC/CB =>
AC = 0,54 * 7 = 3,78. Alternativa (A), portanto.

2 - Podemos escrever seno de 30 como:

sen(30º) = sen(15º + 15º) = sen(15º)cos(15º) + sen(15º)cos(15º) =
2sen(15º)cos(15º)

Dividindo esse valor por sen(15º), que é o termo anterior, obtemos
2cos(15º). Alternativa (D).

2012/3/14 Vanessa Nunes de Souza 

>
> Olá, mas uma vez gostaria da ajuda dos colegas em algumas questões de
> concurso.
>
> 1-João se desloca diariamente de sua casa (ponto C) até o trabalho (ponto
> T), passando pelo (ponto B) em trajetórias retilíneas, conforme mostra a
> figura :
>
> Desenho em anexo
>
> Considere-se que, num determinado dia, João percorreu: - a distância CB,
> com velocidade média de 10 km/h; - a distância BT = 12 km, com velocidade
> média de 15 km/h;
> - toda essa trajetória em 1h30min.
> Se o seno do ângulo CBT vale 0,54, o segmento CA,
> perpendicular à reta AT, mede, em km:
> (A) 3,78
> (B) 3,29
> (C) 2,56
> (D) 2,14
>
> 2-A sequência (sen15º, sen30º,...) é uma progressão geométrica.
> A razão dessa progressão é igual a:
> (A)
> (B) 2
> (C) sen2°
> (D) 2cos15°
>
> Agradeço quem puder ajudar.
> Vanessa Nunes
>


Re: [obm-l] Conta Matematica

2012-03-04 Por tôpico Gabriel Merêncio
>Não concordo que as calculadoras não sejam exatas.

Quando mencionei "(ou deveriam ser) exatas", quis dizer com isso que a
interpretação da entrada não pode ser ambígua. Fica claro que qualquer
forma de computação tem instruções bem definidas... O que não impede
programadores de transmitirem interpretações equivocadas ao projeto.

No caso, espera-se que uma calculadora efetue as operações por ordem de
prioridade e, no caso de prioridades iguais, prossiga da esquerda para a
direita. Qualquer outro procedimento não é uma interpretação literal, exata.

2012/3/4 Chicao Valadares 

> Ambas realizam a operação entre parentesis primeiro. O problema é que uma
> dá preferência a divisão ao invés da multiplicação e a outra dá preferência
> a operação de multiplicação ao invés de divisão quando as duas operações
> aparecem juntas e não há um parentesis indicando quem vai ser realizado
> primeiro. Isso é definido na hora de construir o analisador sintático da
> calculadora. O mesmo ocorre com as outras operações( ex: soma e
> subtração).As linguagens de programação também passam pelo mesmo processo,
> são definidas ordens de precendencia na realização de determinadas
> operações.Não concordo que as calculadoras não sejam exatas. Pelo
> contrário, elas são tão exatas que possuem passos definidos na hora
> realizar os
>  cálculos. O problema que eu vejo aqui são duas calculadoras do mesmo
> fabricante com ordem de realização das operaçoes diferentes.No mínimo para
> cada fabricante a ordem das operações deveria ser a mesma.
>
>
>
>
> "O ideal no casamento é que a mulher seja cega e o homem surdo."
>
> Sócrates
>
>
>
> "O Binômio de Newton é tão belo como a Vênus de Milo.
>
> O que há é pouca gente para dar por isso... "
>
> Fernando Pessoa - Poesias de Alvaro Campos
>
>
>
> _
>
> As informações existentes nessa mensagem e no(s) arquivo(s) anexado(s)
>
> são para uso restrito, sendo seu sigilo protegido por lei. Caso não seja
> destinatário, saiba que leitura, divulgação ou cópia são proibidas.
>
> Favor apagar as informações e notificar o remetente. O uso impróprio será
> tratado conforme as normas da empresa e a legislação em vigor. Agradecemos
> sua colaboração.
>
>
>
> The information mentioned in this message and in the archives attached
>
> are of restricted use, and its privacy is protected by law. If you are not
> the addressee, be aware that reading, disclosure or copy are forbidden.
>
> Please delete this information and notify the sender. Inappropriate use
> will be tracted according to company's rules and valid laws. Thank you for
> your cooperation.
>
> 
>
> --- Em qui, 1/3/12, Gabriel Merêncio 
> escreveu:
>
> De: Gabriel Merêncio 
> Assunto: Re: [obm-l] Conta Matematica
> Para: obm-l@mat.puc-rio.br
> Data: Quinta-feira, 1 de Março de 2012, 10:05
>
> Não seria a primeira vez que falhas envolvendo calculadoras Casio
> acontecem:
>
> http://www.washingtonpost.com/wp-srv/aponline/2226/aponline161058_000.htm
>
> http://media.newtella.de/get/media/ObDDCXkwk8L6gZ9blf8t/525
>
> Enfim, concordo com o Bernardo e não vejo nada de muito polêmico na
> questão. O único modo de obter 1 como resultado é calculando 6/(2(1 + 2)),
> porque é avaliado o que está dentro dos parênteses e depois realizada a
> divisão. Provavelmente o engano é devido à intuição: é comum escrever 6/2(1
> + 2), quando na verdade queremos dizer 6/(2(1 + 2)). Mas máquinas são (ou
> deveriam ser) exatas, com instruções claras e precisas.
>
> 2012/3/1 Bernardo Freitas Paulo da Costa 
>
> 2012/3/1 Luís Lopes :
>
> > Sauda,c~oes,
>
> >
>
> > Num forum familiar o que segue abaixo está pegando fogo.
>
> >
>
> > E estou sendo muito cobrado a dar uma resposta definitiva. :)
>
> >
>
> > Que não possuo.
>
> >
>
> > O debate está aberto aqui mas gostaria de conhecer o histórico
>
> > da questão. Pois se não for armação acho que devem ter
>
> > rolado algumas cabeças no fabricante da calculadora.
>
> Bom, eu não conheço o histórico, mas a minha opinião é que as regras
>
> de preferência de operadores são um tanto explícitas em matemática:
>
> - parênteses, ou qualquer outra forma de grupos com início e fim
>
> (colchetes, chaves, barras, ...)
>
> - exponenciação
>
> - multiplicação e divisão
>
> - adição e subtração
>
>
>
> Quando há duas operações do mesmo nível, resolve-se da esquerda para a
>
> direita. Assim, a + b - c = (a + b) - c, a - b + c = (a - b) + c, a +
>
> b - c - d + e + f - g

Re: [obm-l] Conta Matematica

2012-03-01 Por tôpico Gabriel Merêncio
Não seria a primeira vez que falhas envolvendo calculadoras Casio acontecem:

http://www.washingtonpost.com/wp-srv/aponline/2226/aponline161058_000.htm

http://media.newtella.de/get/media/ObDDCXkwk8L6gZ9blf8t/525

Enfim, concordo com o Bernardo e não vejo nada de muito polêmico na
questão. O único modo de obter 1 como resultado é calculando 6/(2(1 + 2)),
porque é avaliado o que está dentro dos parênteses e depois realizada a
divisão. Provavelmente o engano é devido à intuição: é comum escrever 6/2(1
+ 2), quando na verdade queremos dizer 6/(2(1 + 2)). Mas máquinas são (ou
deveriam ser) exatas, com instruções claras e precisas.

2012/3/1 Bernardo Freitas Paulo da Costa 

> 2012/3/1 Luís Lopes :
> > Sauda,c~oes,
> >
> > Num forum familiar o que segue abaixo está pegando fogo.
> >
> > E estou sendo muito cobrado a dar uma resposta definitiva. :)
> >
> > Que não possuo.
> >
> > O debate está aberto aqui mas gostaria de conhecer o histórico
> > da questão. Pois se não for armação acho que devem ter
> > rolado algumas cabeças no fabricante da calculadora.
> Bom, eu não conheço o histórico, mas a minha opinião é que as regras
> de preferência de operadores são um tanto explícitas em matemática:
> - parênteses, ou qualquer outra forma de grupos com início e fim
> (colchetes, chaves, barras, ...)
> - exponenciação
> - multiplicação e divisão
> - adição e subtração
>
> Quando há duas operações do mesmo nível, resolve-se da esquerda para a
> direita. Assim, a + b - c = (a + b) - c, a - b + c = (a - b) + c, a +
> b - c - d + e + f - g = ( ... (a + b) - c) - d ) + e ) + f ) - g
>
> Mais ainda, o sinal de multiplicação pode ser omitido quando for
> claro. Assim, 45(a+b) = 45 * (a + b).
>
> Portanto, 6/2(1+2) = 6/2*(1+2) = 6/2*3 = (6/2)*3 = 9.
>
> É claro que você poderia tentar justificar o outro resultado dizendo
> que a "multiplicação implícita" tem preferência sobre os demais
> operadores mas não sobre os grupos. Isso não faz nenhum sentido para
> mim, porque inclui um novo tipo de elemento na sua árvore sintática
> (se é que a calculadora implementa uma árvore sintática para analisar
> as expressões, da mesma forma que um compilador faz na hora de
> construir as instruções), além de criar uma regra a mais para decorar.
>
> > Abs,
> > Luís
> >
> > http://9gag.com/gag/2957368
>
>
> --
> Bernardo Freitas Paulo da Costa
>
> =
> Instruções para entrar na lista, sair da lista e usar a lista em
> http://www.mat.puc-rio.br/~obmlistas/obm-l.html
> =
>


[obm-l] RE: [obm-l] Canguru matemático

2012-02-22 Por tôpico Gabriel Franco

como faço para parar de receber emails?

Date: Wed, 22 Feb 2012 10:09:15 -0200
From: douglas.olive...@grupoolimpo.com.br
To: obm-l@mat.puc-rio.br
Subject: Re: [obm-l] Canguru matemático



 
Olá , bom multiplicando a segunda por a+b+C em ambos os lados teremos 
(a+b+c)/a+b +(a+b+c)/b+c +(a+b+c)/a+c =7(a+b+c)/10, e abrindo em fracoes 
parciais o primeiro membro teremos 1+c/a+b +1+a/b+c + 1+b/a+c=49/10, daí a 
resposta  a/(b+c) +b/(a+c) + c/(a+b)=49/10 -3=19/10.
Um abraco do Douglas Oliveira!!!
 
 
On Wed, 22 Feb 2012 03:06:56 +, marcone augusto araújo borges wrote:

Se a+b+c=7 e 1/(a+b) + 1/(b+c)  + 1/(a+c) =7/10,quanto vale a/(b+c) +b/(a+c) + 
c/(a+b)?
 Fiz muitas contas e não cheguei ao resultado.Alguem ajuda?
 Obrigado.

 
  

[obm-l] Re: [obm-l] RE: [obm-l] Re: [obm-l] Permutação circular

2012-02-05 Por tôpico Gabriel Merêncio
Desculpe se minha resolução não foi muito rigorosa, admito que me guiei
mais pela intuição... Pelo visto, com resultados pouco positivos.

Mas João, admitindo a restrição adicional de que dois homens não podem
sentar juntos (ou seja, todo homem senta ao lado de duas mulheres),
acredito que seja possível resolver facilmente por análise combinatória.
Primeiro arranja-se os 5 homens de forma circular (separando cada um por
uma posição vazia), o que pode ser feito de 5!/5 = 4! = 24 maneiras. Uma
esposa pode ocupar 3 das 5 posições vazias, já que 2 ficam ao lado do
marido, então há 3! = 6 maneiras de distribuí-la. Portanto, 24 * 6 = 144
possibilidades no total. Generalizando, a fórmula geral seria f(x) = (x -
1)! * (x - 2)!, sendo x o número de casais.

2012/2/5 João Maldonado 

>
> Não entendi seu raciocínio :(
>
> Fiz  um programa  de computador que  calcula todas as possibilidades da
> função f(x),  para x casais
>
> obtive:
> f(0) = 0
> f(1) = 0
> f(2) = 2
> f(3) = 32
> f(4) =  1488
> f(5) = 112512
>
> Se  considerar  que  a formação horária é  igual a anti-horária, divida
>  ainda por 2
>
> Até  o f(2)  é fácil de se  achar
>
> Aqui  vai todas as 192  (que  é  32*6) possibilidades do f(3)  em  linha
>  (ou  seja ,ignorando a igualdade por rotação e considerando que o primeiro
> termo senta ao lado do último)
> Sendo 0,1  o primeiro casal, 2,3 o  segundo...
>
> ['021435', '021534', '024135', '024153', '024315', '025134', '025143',
> '025314', '031425', '031524', '034125', '034152', '034215', '035124',
> '035142', '035214', '041253', '041352', '042135', '042153', '042513',
> '043125', '043152', '043512', '051243', '051342', '052134', '052143',
> '052413', '053124', '053142', '053412', '120435', '120534', '124035',
> '124053', '124305', '125034', '125043', '125304', '130425', '130524',
> '134025', '134052', '134205', '135024', '135042', '135204', '140253',
> '140352', '142035', '142053', '142503', '143025', '143052', '143502',
> '150243', '150342', '152034', '152043', '152403', '153024', '153042',
> '153402', '203415', '203514', '204135', '204315', '204351', '205134',
> '205314', '205341', '213405', '213504', '214035', '214305', '214350',
> '215034', '215304', '215340', '240315', '240351', '240531', '241305',
> '241350', '241530', '243051', '243150', '250314', '250341', '250431',
> '251304', '251340', '251430', '253041', '253140', '302415', '302514',
> '304125', '304215', '304251', '305124', '305214', '305241', '312405',
> '312504', '314025', '314205', '314250', '315024', '315204', '315240',
> '340215', '340251', '340521', '341205', '341250', '341520', '342051',
> '342150', '350214', '350241', '350421', '351204', '351240', '351420',
> '352041', '352140', '402153', '402513', '402531', '403152', '403512',
> '403521', '405213', '405312', '412053', '412503', '412530', '413052',
> '413502', '413520', '415203', '415302', '420351', '420513', '420531',
> '421350', '421503', '421530', '425031', '425130', '430251', '430512',
> '430521', '431250', '431502', '431520', '435021', '435120', '502143',
> '502413', '502431', '503142', '503412', '503421', '504213', '504312',
> '512043', '512403', '512430', '513042', '513402', '513420', '514203',
> '514302', '520341', '520413', '520431', '521340', '521403', '521430',
> '524031', '524130', '530241', '530412', '530421', '531240', '531402',
> '531420', '534021', '534120']
>
>
> Para  o f(3)  tenho um método  para achar o 32  (muito pouco prático),
>  serve também para qualquer  x, mas depois do f(3) fica quase impossível
>  de se fazer  as coisas sem um computador.
> Tentei achar uma  recursão mas  não consegui  (aliás pela fatoração dos
> resultados,  tal recursão teria que ter muitas  somas já que 112512 por
> exemplo  tem fator 293, ou  seja, provavelmente não seria viável
>
> Para  o caso específico  de homem sentar ao lado de mulher  achei  uma
> recursão   e uma fórmula fácil  (se você entende de  teoria do caos).
>
> []'s
> João
>
> --
> Date: Sun, 5 Feb 2012 16:39:02 -0200
> Subject: [obm-l] Re: [obm-l] Permutação circular
> From: gmerencio.san...@gmail.com
> To: obm-l@mat.puc-rio.br
>
>
> Meu raciocínio é considerar cada casal como uma reta, sendo definida por
> dois pontos: um é o assento do marido e o outro o da esposa. Temos um total
> de C(10, 2) retas para representar o primeiro casal. Para descontar as
> configurações iguais por rotação, dividimos esse número por 5. Finalmente,
> sabemos que exatamente 2 dessas retas são inválidas, pois nelas o marido
> fica ao lado de sua esposa (já descontamos as outras obtidas por rotação) .
>
> Continuando a lógica para os 8 assentos restantes, vemos que agora
> dividimos por 4, já que um casal já está à mesa. Resultados semelhantes
> podem ser inferidos para 6 e depois 4 assentos restantes, só restando 1
> possibilidade para o último par. Portanto:
>
> [C(10,2)/5 - 2] * [C(8,2)/4 - 2] * [C(6,2)/3 - 2] * [C(4,2)/2 - 2] * 1 =
> = 7 * 5 * 3 * 1 * 1 =
> = 105
>
> 2012/1/27 Carlos Gomes 
>
> **
> Olá amigo

[obm-l] Re: [obm-l] Permutação circular

2012-02-05 Por tôpico Gabriel Merêncio
Meu raciocínio é considerar cada casal como uma reta, sendo definida por
dois pontos: um é o assento do marido e o outro o da esposa. Temos um total
de C(10, 2) retas para representar o primeiro casal. Para descontar as
configurações iguais por rotação, dividimos esse número por 5. Finalmente,
sabemos que exatamente 2 dessas retas são inválidas, pois nelas o marido
fica ao lado de sua esposa (já descontamos as outras obtidas por rotação) .

Continuando a lógica para os 8 assentos restantes, vemos que agora
dividimos por 4, já que um casal já está à mesa. Resultados semelhantes
podem ser inferidos para 6 e depois 4 assentos restantes, só restando 1
possibilidade para o último par. Portanto:

[C(10,2)/5 - 2] * [C(8,2)/4 - 2] * [C(6,2)/3 - 2] * [C(4,2)/2 - 2] * 1 =
= 7 * 5 * 3 * 1 * 1 =
= 105

2012/1/27 Carlos Gomes 

> **
> Olá amigos...alguém poderia me ajudr com a questão:
>
> De quantas formas distintas 5 casais podem ser dispostos em torno de uma
> mesa circular, supondo que cada marido não fique ao lado da sua respectiva
> esposa?
>
> (Duas conficurações são consideradas iguais se uma puder ser obtida da
> outroa por um movimento de rotação!)
>
>
> Obrigado,  Cgomes.
>


[obm-l] Equação funcional

2011-11-16 Por tôpico Luan Gabriel

Galera:Determine todas as funções F: R -> R tais que,para todo x real, 
f(x^2+f(y)) = y + f(x)^2 .

[obm-l] Mais um do majorando

2011-10-22 Por tôpico Luan Gabriel

Ae galera fiquei com dúvida em outra do livro:
Num torneio de futebo lcom n times (n<9), cada time joga com cada um dos outros 
exatamente uma vez, e recebe três pontospor vitória, um por empate e zero por 
derrota.Após o torneio, um dos times foi declarado campeão por ter obtido maior 
somade pontos. Curiosamente, esse time foi o time com o menor número de 
vitórias. Determine n.  

RE: [obm-l] Treinamento IME

2011-10-19 Por tôpico Luan Gabriel

Fala Douglas ! Sua solução tem uma pequena incoerência, apesar de ser brilhante 
pelo fato de ver a equação em "a".
A raiz quadrada em reais resulta em um valor positivo,por isso antes de elevar 
ao quadrado tem que certificar que a expressão x^2-2ax+1-a é não negativa. No 
final, acabam aparecendo incoerências em um pedaço da solução : a 
=(2+x^2+x)/2+2x. Resolvendo, uma das soluções é [2a-1+raiz(4a^2+4a-7)]/2. Essa 
solução tem que ser descartada,pois sempre gera na equação inicial valores 
negativos do lado esquerdo (exemplo : a=1 => x=1 , que na equação original gera 
-1=(1)^1/2 ) .
Assim, a solução fica as duas da equação [x^2-x+1]/2x  e a com o sinal "-" da 
expressão anterior, com as restrições: 1) se 1/2ou= -1/2+raiz(2) , então a solução são 
as anteriores mais a solução [2a-1-raiz(4a^2+4a-7)]/2 .
Consegui outra solução muito boa vendo que a função y= x^2-2ax+1 e y1= a + 
raiz(a^2+x-1) são inversas uma da outra, e o encontro ocorre quando y=y1=x ,mas 
por ela não encontrei a parte 2) da solução.
Abraços,Luan Gabriel 

Date: Tue, 18 Oct 2011 14:52:11 -0200
From: douglas.olive...@grupoolimpo.com.br
To: obm-l@mat.puc-rio.br
Subject: Re: [obm-l] Treinamento IME



Olá joão, bom consegui a número 8, elevando ao quadrado ambos os lados da 
equacao , e escrevendo uma equacao do segundo grau em "a", verá que o delta é 
um quadrado perfeito, vai descobrir que a raiz do delta será:  2(x^2+2x-1), ai 
é só resolver e simplificar as raizes, vai encontrar para solucao as seguintes 
raizes:
a=(2+x^2+x)/2+2xe a outra a=(x^2-x+1)/2x  bom ai agora é resolver as 
equacoes em x e para que as raizes sejam reais, delta maior ou igual a zero em 
ambas, ai vera pelas intercessoes das solucoes que "a" deve satisfazer 
a<(-1-2raiz(2))/2   ou a>(-1+2raiz(2))/2, na verdade era pra ser maior menor ou 
igual, é ue não sei como escrever esse simbolo por aqui!!
Um abraco:
/Douglas Oliveira
 
On Tue, 18 Oct 2011 00:50:03 -0200, João Maldonado wrote:


Boa Tarde,  
O site rumo ao ITA oferece  várias provas para treinamento IME/ITA  (mas 
infelizmente sem resoluções =D)
Não consegui  fazer a questão 8  e nove,  alguém  pode me ajudar?

Questão 8
Resolva a equação x² - 2ax +1 - a = (a² + x - 1)^(1/2) (soluções reais) e 
determine os
valores de a para que essas soluções existam.

Questão 9
Prove  que para uma função f(x)  de concavidade para  baixo,  a função da  
média é maior ou igual à média das funções,  isto é:
f((a+b+c)/3)>=(f(a)+f(b)+f(c))/3
O link está abaixo  para visualisar melhor
http://www.rumoaoita.com/site/attachments/044_simulado_2006_1.pdf
[]'s
João


 
  

[obm-l] RE: [obm-l] Re: [obm-l] RE: [obm-l] RE: [obm-l] polinômios

2011-10-13 Por tôpico Luan Gabriel


Nossa, genial ! Era a última do tópico fatoração de polinômios do majorando,não 
sei de onde ele tirou mas estive batendo muita cabeça nela.
Obrigado =] Abraços,Luan Gabriel

> Date: Thu, 13 Oct 2011 22:25:39 +0200
> Subject: [obm-l] Re: [obm-l] RE: [obm-l] RE: [obm-l] polinômios
> From: bernardo...@gmail.com
> To: obm-l@mat.puc-rio.br
> 
> 2011/10/13 Luan Gabriel :
> > Sem querer ser chato,mas ainda
> > sobrou mais uma questão desse tipo,mas não consegui resolver:
> >
> > Prove que se P(x) tem coeficientes inteiros, então P(x^4).P(x^3).P(X^2).P(x)
> > +1 não possui raízes inteiras.
> 
> Bom, tentando resolver Q(x) = 0, você chega a
> P(x^4) P(x^3) P(x^2) P(x) = -1
> 
> Como você tem um produto de inteiros que dá -1, você precisa que todos
> eles sejam 1 ou -1. E, inclusive, um número ímpar de -1. Essa simples
> observação mostra que x != 0, 1 e -1, porque teríamos o produto de
> dois quadrados (>= 0) à esquerda.
> 
> Bom, a idéia é tentar achar uma contradição, por exemplo achando um
> dos caras de módulo maior do que 1. Eu consegui assim:
> 
> P(x^2) - P(x) = Soma a_n (x^2n - x^n) = Soma a_n x^n (x^n - 1) = (x-1)
> * Soma a_n x^n (x^(n-1) + ... + x + 1)
> 
> Como x é diferente de 1, temos duas possibilidades:
> P(x) = P(x^2), e daí a Soma = 0.
> P(x) = -P(x^2) e daí temos (x-1) * Soma = +- 2.
> 
> Repare que esse mesmo argumento serve para P(x^3) - P(x) e P(x^4) -
> P(x), sendo que o fator que sobra passa a ser
> x^2 - 1 => porque x^(3n) - x^n = x^n (x^(2n) - 1) = x^n (x^2 -
> 1)(x^(2n - 2) + ... + x^2 + 1)
> x^3 - 1 => porque x^(4n) - x^n = x^n (x^(3n) - 1) = x^n (x^3 -
> 1)(x^(3n - 3) + ... + x^3 + 1)
> 
> Agora, repare que como x != 0, 1 e -1, os fatores x^2 - 1 e x^3 - 1
> são maiores do que 2. Assim, não podemos ter P(x) = -P(x^3) nem P(x) =
> -P(x^4). Senão, a diferença seria 2 ou -2, mas seria também divisível
> por x^2 - 1 ou x^3 - 1 que são maiores do que 2.
> 
> Portanto, P(x) = P(x^3) = P(x^4).
> 
> Mas agora faça a mesma coisa para P(x^4) - P(x^2). Dá a mesma coisa
> que P(x^2) - P(x), com x trocado por x^2. Portanto, essa diferença
> também é divisível por x^2 - 1. Que continua sendo maior do que 2. O
> que quer dizer que P(x^2) = P(x^4). Mas daí temos o produto de 4
> fatores iguais, isso não dá um número negativo.
> 
> Ufa!
> 
> Abraços,
> -- 
> Bernardo Freitas Paulo da Costa
> 
> =
> Instruções para entrar na lista, sair da lista e usar a lista em
> http://www.mat.puc-rio.br/~obmlistas/obm-l.html
> =
  

RE: [obm-l] Questao de probabilidade: o sapo e a mosca

2011-10-13 Por tôpico Luan Gabriel

João, Acho que você errou na contagem do caso 4. Você está contando a 
probabilidade da mosca escapar, logo o caso 4 é um caso importante, já que a 
mesma será 1. Mas a contagem da probabilidade do sapo ter comido 3 moscas é a 
seguinte: 
1) A terceira mosca comida é a 5º. Então a probabilidade será C(4,2).(1/2)^52) 
A terceira mosca comida é a 4º : C(3,2).(1/2)^43) A terceira mosca comida é a 
3º : C(2,2).(1/2)^3
 Essa diferença aconteçe porque quando chega na terceira mosca,a probailidade 
das moscas seguintes , tanto a 6º como a 5º ou a 4º se igualam a 1. 
From: joao_maldona...@hotmail.com
To: obm-l@mat.puc-rio.br
Subject: RE: [obm-l] Questao de probabilidade: o sapo e a mosca
Date: Thu, 13 Oct 2011 12:28:33 -0300









Olá RalphNa verdade eu interpretei assim:  Um  sapo come apenas 3 moscas por 
dia,  se ele ainda não comeu as 3 e  uma mosca passa  perto do sapo, é de 50% a 
chance dela sobreviver e 50% de virar alimento.  Caso o  sapo já tenha comido 
as 3 moscas é de 0% a chance dela virar alimento, ela sempre vai  escapar. (Bom 
até aí a gente  tem a mesma opinião)
O que não entendi foi a"ii)  O sapo decide comê-la, mas ele já comeu PELO MENOS 
3 antes" Não sei se dá para ele comer mais de 3 moscas  por dia (claro,  
interpretando que toda a ação aconteceu no mesmo dia).
Neste caso ficaria

1) Se ele comeu 0 moscas:Probabilidade   de ele ter comido 0 mosca: (1/2)^5 . 
C(5, 0)Probabilidade de ele  escapar: (1/2)

2) Se ele comeu 1 mosca:Probabilidade   de ele ter comido 1 mosca: (1/2)^5 . 
C(5, 1)Probabilidade de ele escapar: (1/2)
3) Se ele comeu 2 moscas:Probabilidade   de ele ter comido 2 moscas: (1/2)^5 . 
C(5, 2)Probabilidade de ele ser escapar: (1/2)
4) Se ele comeu 3 moscas:Probabilidade   de ele ter comido 3 moscas: (1/2)^5 . 
C(5, 3)Probabilidade de ele ser  escapar: 1
P = 9/16
Mas para  este acontecimeto,  ao  achar a probabilidade DA MOSCA  SER COMIDA,  
temos que  o caso 4 se iguala a 0,  e a P fica 4/16,  que somando a primeira 
probabilidade temos 13/16   que é diferente de 1
Ainda não sei se  isso é possível, já que  ou ela é comida ou não.Se não for,  
aonde eu errei?

[]'sJoão

Date: Thu, 13 Oct 2011 10:23:24 -0300
Subject: Re: [obm-l] Questao de probabilidade: o sapo e a mosca
From: ralp...@gmail.com
To: obm-l@mat.puc-rio.br

Entendi o enunciado assim: um sapo, *quando faminto*, tem 50% de chance de 
comer uma mosca que passe por perto dele. Ele só deixa de estar faminto após 
comer 3 moscas -- depois da 3a, é 0% de chance de ele comer a mosca. Digo isso 
porque, tecnicamente, não é verdade que "uma mosca tenha 50/50 de chance de 
escapar", pois isto depende de quando a mosca chega.
 Bom, então imagino primeiro um sapo que está sempre faminto... Há duas 
hipóteses excludentes para a 6a mosca escapar:i) O sapo decide não comê-la: 
p1=50%ii) O sapo decide comê-la, mas ele já comeu PELO MENOS 3 antes: 
p2=(Pr(X>=3)).50%
onde X é o número de moscas que este sapo decidiu comer dentre as 5 primeiras 
(que seria uma distribuição binomial cumulativa). Ou 
seja:Pr(X>=3)=C(5,3).(1/2)^5+C(5,4).(1/2)^5+C(5,5).(1/2)^5=1/2
(Faz sentido pela simetria do problema: das 5 primeiras, a probabilidade de ele 
comer 0,1 ou 2 moscas é idêntica à probabilidade de ele comer 3,4 ou 5 moscas.) 
Então no fim das contas: Pr(6a mosca escapar)=1/2+1/4=3/4=75%
 Abraço, Ralph
2011/10/13 Rafael Forte 

Uma mosca passando próxima a um sapo tem 50% de chances de escapar e 50% de 
virar alimento do sapo. Mas o sapo come apenas 3 moscas por dia. Qual é a 
probabilidade da sexta mosca a passar próxima ao sapo sobreviver?


Abracos,

Rafael


  

  1   2   3   4   >